Psychiatry Test Preparation and Review Manual, 7 edition

Chapter 4: Test Number Four

1. Which one of the following is false regarding Freud’s theories of human development?

   A During development, sexual energy shifts to different areas of the body that are usually associated with eroticism

   B The anal phase is from age 1 year to 3 years

   C Latency is marked by a sharp increase in sexual interest

   D Freud thought that resolution of his stages was essential to normal adult functioning

   E The phallic stage is from age 3 to 5 years

2. Ischemia to which one of the following arterial territories is responsible for the phenomenon known as amaurosis fugax?

   A Carotid

   B Vertebrobasilar

   C Lenticulostriate

   D Anterior cerebral

   E Middle cerebral

3. Which one of the following is true regarding norepinephrine (NE) and/or the locus caeruleus?

   A NE is synthesized in the locus caeruleus

   B Dopamine is synthesized in the locus caeruleus, NE in the dorsal raphe nuclei

   C Acetylcholine is synthesized with NE in the substantia nigra

   D GABA is synthesized in the locus caeruleus

   E The locus caeruleus is the site of formation of serotonin

4. A 75-year-old man with a recent history of influenza vaccination presents to the emergency room with an acute onset of paraparesis and urinary incontinence. He reported that his symptoms began a week earlier with dull, progressive low back pain that soon resulted in bilateral leg weakness. The most likely diagnosis in this case is:

   A Spinal cord metastases

   B Acute spinal cord compression

   C Vacuolar myelopathy

   D Transverse myelitis

   E Acute disseminated encephalomyelitis

5. A doctor in a certain hospital makes a diagnosis for a particular patient. That diagnosis is considered reliable if:

   A It is accurate

   B Many different doctors in different locations would agree upon the same diagnosis

   C The disorder has features characteristic enough to distinguish it from other disorders

   D The disorder allows doctors to predict the clinical course and treatment response

   E The diagnosis is based on an understanding of the underlying pathophysiology and has biological markers

6. Which one of the following set of symptoms and signs correctly identifies Horner’s syndrome?

   A Ptosis, miosis, sweating

   B Ptosis, mydriasis, sweating

   C Ptosis, miosis, anhydrosis

   D Ptosis, mydriasis, anhydrosis

   E Lid-lag, miosis, anhydrosis

7. A patient presents with the following symptoms: tremor, halitosis, dry mouth, tachycardia, hypertension, fever, euphoria, alertness, agitation, paranoia, hallucinations, and irritability. Which one of the following substances is the most likely cause?

   A Amphetamines

   B Opioids

   C Alcohol

   D Barbiturates

   E Benzodiazepines

8. The inability to recognize familiar faces is known as:

   A Anosognosia

   B Simultanagnosia

   C Aprosodia

   D Prosopagnosia

   E Astereognosis

9. Which one of the following is not a potential effect of PCP use?

   A Paranoia

   B Nystagmus

   C Catatonia

   D Convulsions

   E Hypotension

10. A characteristic of a facial nerve (Bell’s) palsy that clearly distinguishes it from a stroke-related facial paresis is:

   A The presence of miosis

   B The involvement of the whole face

   C Anhydrosis

   D Rapid recovery of motor functioning

   E A post-infectious onset

11. Object permanence develops during which one of Piaget’s developmental stages?

   A Sensorimotor

   B Preoperational thought

   C Concrete operations

   D Formal thought

   E Rapprochement

12. Which one of the following is not a classic feature of syringomyelia?

   A Spasticity

   B Muscular atrophy

   C Fasciculations

   D Loss of temperature and pain sensation

   E Preservation of proprioception

13. A patient comes into the emergency room and admits to sniffing glue daily for the past eight months. Which one of the following is not your concern as this patient’s physician?

   A Liver damage

   B Permanent brain damage

   C Kidney damage

   D Myocardial damage

   E Urinary retention

14. One of the most common causes of the movement disorder of opsoclonus–myoclonus in the infant is:

   A Neonatal seizures

   B Craniopharyngioma

   C Prematurity

   D Neuroblastoma

   E Meningioma

15. A physician examines a patient in the emergency room who has recently been diagnosed with a social phobia. Which one of the following answer choices would most likely be the greatest fear for this patient?

   A Having to take responsibility for planning a dinner with her husband

   B Being in a relationship with a new boyfriend

   C Going to a state fair and being around thousands of people

   D Being scrutinized by others

   E Competing for a new position that just opened up in her company

16. The Ramsay Hunt syndrome classically affects which one of the following pairs of cranial nerves?

   A III and VI

   B IV and VI

   C V and VII

   D V and VIII

   E VII and VIII

17. Which one of the following statements is not true regarding body dysmorphic disorder?

   A It is a preoccupation with an imagined defect in appearance

   B It is most commonly associated with a comorbid mood disorder

   C The most common concerns involve facial flaws

   D Treatment with surgical, medical, dental or dermatological care is usually successful

   E If a slight physical anomaly is present the concern is markedly excessive

18. Which one of the following primary brain tumors is the most common in patients over 60 years of age?

   A Anaplastic astrocytoma

   B Glioblastoma multiforme

   C Meningioma

   D Ependymoma

   E Acoustic neuroma

19. Patients with obsessive–compulsive disorder are noted to have anomalies of which one of the following brain regions?

   A Corpus callosum

   B Striatum

   C Hippocampus

   D Caudate nucleus

   E Cerebellum

20. Anton’s syndrome results from a stroke that localizes to the:

   A Frontal lobes

   B Temporal lobes

   C Parietal lobes

   D Occipital lobes

   E Cerebellar hemispheres

21. Which one of the following should not be considered a predictive factor for violence?

   A Alcohol intoxication

   B Recent acts of violence

   C Command auditory hallucinations

   D High socioeconomic status

   E Menacing other people

22. Riluzole, the only agent FDA-approved in the treatment of amyotrophic lateral sclerosis, affects which one of the following neurotransmitters?

   A Glutamate

   B GABA

   C Acetylcholine

   D Dopamine

   E Norepinephrine

23. A 29-year-old patient is admitted to the neurology unit for evaluation of seizures. Workup is negative and there has not been any seizure activity captured on electroencephalogram during his seizure episodes. Neurological examination is negative. The patient is noted to be very sad, and a psychiatric consult is called. Nurses have noted conflict between the patient and his wife during her visits. As the consultant, which one of the following would be the highest on your list of differential diagnoses?

   A Obsessive–compulsive disorder

   B Conversion disorder

   C Somatization disorder

   D Social phobia

   E Panic disorder

24. What is the most common CNS cancer noted in patients with advanced AIDS?

   A Glioblastoma multiforme

   B Lymphoma

   C Meningioma

   D High-grade brainstem glioma

   E Epidermoid tumor

25. The highest rate of synapse formation in the brain takes place during which one of the following time periods?

   A Adolescence

   B Weeks 32 to 35 of gestation

   C Weeks 13 to 26 of gestation

   D Within the first 6 weeks of gestation

   E As a toddler

26. Which one of the following is not a presenting feature of AIDS dementia complex?

   A Poor attention and concentration

   B Slowness of thinking

   C Personality changes

   D Apathy

   E Hemiparesis

27. Which one of the following is not a medical complication of weight loss in eating disorders?

   A Cachexia

   B Loss of cardiac muscle

   C Delayed gastric emptying

   D Lanugo

   E Increased bone density

28. The most frequent opportunistic CNS infection in the AIDS patient is:

   A CNS toxoplasmosis

   B Cryptococcal meningitis

   C Herpes meningitis

   D Cytomegalovirus encephalitis

   E Neurosyphilis

29. Which one of the following is not a medical complication of purging seen in eating disorders?

   A Electrolyte abnormalities

   B Salivary gland inflammation

   C Erosion of dental enamel

   D Hyperkalemia

   E Seizures

30. A 45-year-old man with end-stage AIDS and CD4 count of 50/μL presents to the ER with a complaint of rapidly progressive onset of gait difficulty, spasticity, leg weakness, sphincter dysfunction and loss of proprioception to both feet and legs. The most likely diagnosis in this case is:

   A Progressive multifocal leukoencephalopathy

   B Distal sensory polyneuropathy

   C Vacuolar myelopathy

   D Neurosyphilis

   E HTLV-1 myelopathy

31. Which one of the following medical conditions should not be considered when evaluating patients with anxiety disorders?

   A Carcinoid syndrome

   B Hyperventilation syndrome

   C Hypoglycemia

   D Hyperthyroidism

   E Central serous chorioretinopathy

32. A 45-year-old man with end-stage AIDS and CD4 count of 0/μL presents to the ER with a complaint of progressive onset over the past few weeks of ataxia, visual field deficits, altered mental status, aphasia and fluctuating sensory deficits. T2-weighted brain MRI reveals the image to the right. The most likely diagnosis in this case is:

   A CNS toxoplasmosis

   B Lymphoma

   C Neurosyphilis

   D AIDS dementia complex

   E Progressive multifocal leukoencephalopathy

image

33. When diagnosing a patient with social phobia, which axis II diagnosis is it most important for the treating psychiatrist to keep in mind?

   A Paranoid personality disorder

   B Schizoid personality disorder

   C Obsessive–compulsive personality disorder

   D Avoidant personality disorder

   E Borderline personality disorder

34. Which one of the following is not a prion disease?

   A Kuru

   B Gerstmann–Sträussler–Scheinker syndrome

   C Fatal familial insomnia

   D Devic’s syndrome

   E Creutzfeldt–Jakob disease

35. A psychiatrist is covering the emergency room and a patient comes in who has a previous diagnosis of bipolar II disorder. Based on this diagnosis which one of the following symptoms would the knowledgeable psychiatrist expect to see in this patient over time?

   A Psychotic features

   B Manic episodes that do not respond to treatment with mood stabilizers

   C Rapid cycling between severe depression and mania

   D Recurrent manic episodes in the absence of depression

   E Recurrence of both depressive and hypomanic episodes

36. The classic triad of headache, ipsilateral Horner’s syndrome and contralateral hemiparesis is generally due to:

   A Carotid artery occlusion

   B Giant cell arteritis

   C Wallenberg’s syndrome

   D Pontine hemorrhage

   E Cerebral aneurysmal rupture

37. A set of statistical procedures designed to compare two or more groups of observations and determine whether the differences are due to chance or experimental difference is called:

   A Correlation coefficient

   B Control group

   C Analysis of variance (ANOVA)

   D Regression analysis

   E Null hypothesis

38. A 55-year-old woman presents to the emergency room with a sudden and acute onset of right-sided painless complete facial paralysis, involving both the upper and lower parts of the face. The symptoms began earlier that day and were initially accompanied by a mild ache behind the ear which resolved. The organism most likely responsible for this condition is:

   A Borrelia burgdorferi

   B Herpes simplex virus

   C Epstein–Barr virus

   D West Nile virus

   E Varicella zoster virus

39. A psychiatrist is asked to evaluate a child who does not make appropriate eye contact, fails to respond to the social cues of others, lacks the ability for spontaneous make-believe play, and on close examination has a delay in language development. What is this child’s most likely diagnosis?

   A Schizophrenia

   B Avoidant personality disorder

   C Asperger’s disorder

   D Conduct disorder

   E Autism

40. A veterinary radiologist presents to the emergency room with a 4-week history of headache, vague fever, and paresthesias in the fingers and toes. His temperature is 103.5°F (39.7 °C). He complains of difficulty swallowing with pharyngeal spasms for the past 3 days. The most likely diagnosis is:

   A West Nile virus infection

   B Tetanus infection

   C Acute botulism

   D Rabies

   E Epstein–Barr virus infection

41. Which one of the following receptor subtypes is associated with the neurotransmitter glutamate?

   A Nicotinic

   B Muscarinic

   C Alpha 1

   D AMPA

   E GABA

42. A 60-year-old woman presents to the emergency room with a progressive downward course over the past 6 months characterized by behavioral disinhibition, emotional lability, severe naming and word-finding difficulties, hyperorality, stubbornness, inability to plan, and poor judgment. Autopsy of this patient’s brain would most likely reveal:

   A Hirano bodies

   B Pick’s inclusion bodies and gliosis

   C Lewy bodies

   D Neurofibrillary plaques and tangles

   E Severe white-matter demyelination

43. Your friend just had a baby that is 8 months old. You and she talk about the child and note its temperament. At this point in the child’s development you tell your friend that the child’s temperament is most likely a function of:

   A Biological factors

   B The parent’s culture

   C The grandmother’s influence on weekends

   D The baby’s birth month

   E The influence of the child’s siblings

44. The neuropathological hallmark of idiopathic Parkinson’s disease is:

   A Brainstem Lewy bodies

   B Hirano bodies

   C Amyloid plaques

   D Mesial temporal sclerosis

   E Caudate nucleus atrophy

45. A young woman comes to a psychiatrist’s office seeking help because of problems on her job. She describes nervousness talking in front of other coworkers at conferences, and difficulty at social events. She thinks that her boss knows her inner feelings and that there was wording put in everyone’s contracts with her specifically in mind. Her dress is eccentric and out of date. She complains that she does not have any friends at the office. Given this picture, she most likely has which one of the following diagnoses?

   A Borderline personality disorder

   B Dependent personality disorder

   C Schizotypal personality disorder

   D Histrionic personality disorder

   E Schizoid personality disorder

46. Which one of the following is not a possible symptom of fibromyalgia?

   A Headache

   B Psychosis

   C Depression

   D Sleep disturbance

   E Paresthesias

47. Pancreatic cancer is most often associated with which one of the following psychiatric disorders?

   A Psychosis

   B Anxiety

   C Depression

   D Impulse control disorders

   E Bulimia

48. A 75-year-old man with known history of prostate cancer presents to the emergency room with an acute onset of bilateral leg weakness, leg spasticity, sensory loss to pain and temperature below the waist, and acute bladder and bowel incontinence. The first test of choice to perform in the emergency room setting is:

   A Noncontrast head CT

   B Brain MRI

   C Screening spine MRI

   D Spinal X-rays

   E HTLV-1 antibody titer

49. Which one of the following symptoms would a psychiatrist look for in a child to make the diagnosis of conduct disorder rather than depression, ADHD, or bipolar disorder?

   A Irritable mood

   B Difficulty organizing tasks

   C Excessive activity

   D Starting fights with other children

   E Sleep disturbance

50. Subacute combined degeneration of the spinal cord is a result of deficiency in which one of the following:

   A Vitamin B12

   B Vitamin B1

   C Vitamin B6

   D Niacin

   E Folic acid

51. When patients who have been victim to childhood incest become adults, which one of the following disorders are they most prone to develop?

   A Anorexia

   B Bipolar disorder

   C Social phobia

   D Major depression

   E Conversion disorder

52. A patient presents to the emergency room with an acute onset of pure right hemiparesis that on examination is noted to affect the face, arm and leg equally. There is no sensory deficit, and no cortical signs are noted. This stroke most likely localizes to the:

   A Left thalamus and internal capsule

   B Left internal capsule

   C Right basis pontis

   D Right medulla

   E Left midbrain

53. Which one of the following statements is true regarding the amino acid neurotransmitters?

   A Histamine is an amino acid neurotransmitter

   B GABA is an excitatory amino acid neurotransmitter

   C Glutamate is an inhibitory amino acid neurotransmitter

   D Glutamate receptors have been found to be important in the mechanism of action for cocaine

   E Benzodiazepines, barbiturates and several anticonvulsants work through mechanisms involving GABA-B

54. Which one of the following is not a symptom or sign of Parkinson’s disease?

   A Bradykinesia

   B Loss of postural reflexes

   C Tremor

   D Choreoathetosis

   E Rigidity

55. A psychiatrist is asked to evaluate an 8-year-old girl. She does not want to go to school, and refuses to do her homework. Her teacher reports that she will not read out loud in class. She likes to go and spend the weekends at friends’ houses or go on over-night trips with her grandparents. Her IQ is average. What diagnosis should the informed psychiatrist most strongly consider?

   A ADHD

   B Conduct disorder

   C Separation-anxiety disorder

   D Pervasive developmental disorder

   E Reading disorder

56. Which one of the following is not a feature of botulism toxin poisoning?

   A Myoclonus

   B Dysphagia

   C Diplopia

   D Nausea

   E Urinary retention

57. A couple brings their son in to see a psychiatrist. The child fights with his mother and father and is rude and dismissive toward them. He states that he wants to leave, and when the doctor tells him he has to stay he yells, curses, cries, and rolls around on the floor. His teacher tells the psychiatrist that his work at school is good, but that he gets very nasty with her when she tells him to do a particular task, and he often refuses to cooperate with her. The child’s most likely diagnosis is:

   A ADHD

   B Bipolar disorder

   C Conduct disorder

   D Oppositional defiant disorder

   E Separation-anxiety disorder

58. Balint’s syndrome is the result of a lesion to which one of the following areas?

   A Frontal lobe

   B Temporal lobe

   C Parietal lobe

   D Occipital lobe

   E Bilateral parietal–occipital lobes

59. A psychiatrist evaluates a 7-year-old patient who is brought in by his parents because of complaints they have been receiving from school. The child has been sexually provocative with other children, sexualizes play activities, and openly displays sexual behavior. The most likely cause of this behavior is:

   A Normal development

   B Early onset puberty

   C Traumatic brain injury

   D Sexual abuse

   E Psychosis with sexual delusions

60. Which one of the following symptoms is a feature of Anton’s syndrome?

   A Aphasia

   B Prosopagnosia

   C Apraxia

   D Confabulation

   E Hiccups

61. Which one of the following personality disorders is least associated with violent behavior?

   A Borderline personality disorder

   B Histrionic personality disorder

   C Narcissistic personality disorder

   D Dependent personality disorder

   E Antisocial personality disorder

62. Inhalant intoxication (sniffing glue) causes which one of the following neurologic conditions?

   A Myopathy

   B Neuropathy

   C Myelopathy

   D Denervation of muscle

   E Seizures

63. A 30-year-old man presents to the emergency room with a complaint that there is a cockroach living in his rectum. He says he knows that there is a hole in the side of his rectum which was created by the roach. He has no prior psychiatric history, but states that he has felt the roach crawling around in his rectum for the past 18 months. He is a lawyer with a busy successful practice, and says he has no problems at work. During previous trips to the doctor for this complaint, he was examined and told that there was no cockroach in his rectum. He feels that they just did not examine him properly, otherwise the roach would have been found. Examination and blood work are all normal. His most likely diagnosis is:

   A Conversion disorder

   B Schizophrenia

   C Depression with psychotic features

   D Delusional disorder with somatic features

   E Hypochondriasis

64. Which one of the following substances of abuse is the most likely to lower the seizure threshold during intoxication?

   A Morphine

   B PCP

   C Cocaine

   D Cannabis

   E Alcohol

65. Five patients are brought into the emergency room on a Friday evening. Of the five, which one is most likely to kill themselves successfully?

   A Bob, who has schizophrenia

   B Carol, who has alcoholism

   C Dave, who is mentally retarded

   D Sally, who has borderline personality disorder

   E Mark, who has major depressive disorder

66. Meige’s syndrome comprises which one of the following sets of symptoms?

   A Hemifacial spasm and seizures

   B Hemifacial spasm and cervical dystonia

   C Blepharospasm and ptosis

   D Blepharospasm and oromandibular dystonia

   E Lid apraxia and myokymia

67. A physician that has reason to believe a patient may kill or injure another person must notify the potential victim, authorities, or the victim’s family or friends. This is the result of which one of the following answer choices?

   A Durham rule

   B M’Naghten rule

   C Ford vs Wainwright

   D Tarasoff rule

   E Respondeat superior

68. The phenomenon of scanning speech results from a lesion to the:

   A Cerebellum

   B Thalamus

   C Frontal lobes

   D Midbrain

   E Dominant temporal lobe

69. A friend just gave birth to a healthy baby boy 5 days ago. Now your friend is crying and irritable, and has been dysphoric over the past 2 days. Which one of the following is the most likely diagnosis based on the information given?

   A Postpartum depression

   B Postpartum blues

   C Postpartum psychosis

   D Postpartum bipolar disorder

   E Specific phobia of being a parent

70. Which one of the following is a syndrome of near muteness with normal reading, writing and comprehension?

   A Aphasia

   B Apraxia

   C Agnosia

   D Aphemia

   E Abulia

71. Which one of the following is not true regarding the N-methyl-D-aspartate (NMDA) receptor?

   A The NMDA receptor has been linked with learning and memory

   B The NMDA receptor allows for the passage of potassium only

   C The NMDA receptor only opens when it has bound two molecules of glutamate and one molecule of glycine

   D The NMDA receptor can be blocked by physiological concentrations of magnesium

   E The NMDA receptor can be blocked by PCP

72. Which one of the following refers to a state of unresponsiveness from which arousal only occurs with vigorous and repeated stimulation?

   A Alertness

   B Lethargy

   C Stupor

   D Coma

   E Persistent vegetative state

73. Of the following disorders, which one has the best prognosis?

   A Somatization disorder

   B Body dysmorphic disorder

   C Pain disorder

   D Hypochondriasis

   E Conversion disorder

74. Which one of the following statements is true?

   A All patients with acute Guillain–Barré syndrome should be hospitalized in an intensive care unit in case of respiratory compromise

   B All comatose patients require a head CT scan before a lumbar puncture is performed

   C A positive grasp reflex is always a sign of frontal lobe damage

   D Cerebellar hemispheric lesions produce deficits that are contralateral to the lesion

   E Bell’s palsy is most often caused by Borrelia burgdorferi infection

75. Which one of the following choices is considered unethical by the American Psychiatric Association Ethics Committee?

   A Closing a practice and finding follow-up care for your patients

   B Refusing to discuss a patient’s case with her family unless she gives you permission

   C Charging a colleague rent to sublet office space from you

   D A patient wills you their estate after death. You accept and use the money for a new car

   E You charge a fee to supervise another psychiatrist

76. Emotional memory localizes to the:

   A Amygdala

   B Hippocampus

   C Primary auditory cortex

   D Nucleus basalis of Meynert

   E Pons

77. Of the following medications, which one is the least anticholinergic?

   A Amitriptyline

   B Imipramine

   C Desipramine

   D Nortriptyline

   E Maprotiline

78. By what age should a child have a six-word vocabulary, be able to self-feed, and be able to walk up steps with his hand being held?

   A 6 months

   B 9 months

   C 12 months

   D 18 months

   E 24 months

79. A 30-year-old male is brought to the emergency room after being arrested for exposing his genitals to women on the train. He states that he has impulses to expose himself that he can’t control, and that he finds the whole experience very sexually exciting. Which one of the following medications would be an appropriate treatment for this patient?

   A Medroxyprogesterone acetate

   B Lorazepam

   C Ziprasidone

   D Duloxetine

   E Chlorpromazine

80. A 45-year-old woman presents to the emergency room with an acute left hemiparesis of the arm and leg. You ask her to lift her normal right leg while you put your hand under her paretic leg. You note that while lifting her good leg, she pushes her affected leg downwards on the bed with normal strength. You suspect a hysterical or psychogenic disorder. This phenomenon is termed:

   A Hoffman’s sign

   B Hoover’s sign

   C Lasègue’s sign

   D Romberg sign

   E Gegenhalten

81. Clomipramine is not used for which one of the following?

   A Depression

   B Obsessive–compulsive disorder

   C Panic disorder

   D Premature ejaculation

   E Command auditory hallucinations

82. The only true emergency in neurology that requires immediate MRI imaging and evaluation in the emergency room is:

   A Acute suspected early hemispheric stroke

   B Acute suspected myasthenia gravis.

   C Acute suspected spinal cord compression

   D Acute suspected Guillain–Barré syndrome

   E Acute suspected subarachnoid hemorrhage

83. Which one of the following terms refers to the state’s right to intervene and act as a surrogate parent for those who cannot care for themselves?

   A Actus reus

   B Mens rea

   C Parens patriae

   D Durable power

   E Respondeat superior

84. Which one of the following medications has the unique mechanism of action of being a selective GABA reuptake inhibitor?

   A Gabapentin

   B Tiagabine

   C Pregabalin

   D Vigabatrin

   E Lioresal

85. Which one of the following answer choices is measured by the trail making test?

   A Memory

   B Language

   C Social learning

   D Psychosis

   E Executive function

86. The principal mechanism of action of the Alzheimer’s disease agent memantine involves which one of the following receptors?

   A Acetylcholine

   B Dopamine

   C NMDA

   D Glycine

   E GABA

87. Which one of the GABA receptors is thought to be the site of action of the benzodiazepines?

   A GABA-A

   B GABA-B

   C GABA-C

   D GABA-D

   E GABA-E

88. Vagal nerve stimulation is FDA-approved for which one of the following indications?

   A Refractory epilepsy

   B Bipolar mania

   C Schizophrenia

   D Intermittent explosive disorder

   E Obsessive–compulsive disorder

89. Which one of the following statements is true regarding carbamazepine?

   A Carbamazepine is approved in the US for treatment of temporal lobe epilepsy and general epilepsy, but not trigeminal neuralgia

   B Carbamazepine is metabolized by the kidneys

   C Carbamazepine can be associated with a transient increase in the white blood cell count

   D Carbamazepine has been shown to be as effective as the benzodiazepines in some studies for management of alcohol withdrawal

   E A benign pruritic rash occurs in 60–70% of patients treated with carbamazepine

90. Auscultation of the head that reveals a bruit would likely be indicative of which one of the following?

   A Brain tumor

   B Venous sinus thrombosis

   C Temporal arteritis

   D Intracranial aneurysm

   E Arteriovenous malformation

91. Which one of the following tests is considered to be projective?

   A Halstead–Reitan battery

   B Stanford–Binet test

   C Wechsler–Bellevue test

   D Draw-a-Person test

   E MMPI

92. A 47 year-old man presents to the emergency with intermittent headaches, and periodic drop attacks. His brain MRI on T1 weighted imaging reveals the scan to the right. The most likely diagnosis is:

   A Choroid plexus papilloma

   B Colloid cyst of the third ventricle

   C Ependymoma

   D Pineal region germinoma

   E Pituitary macroadenoma

image

93. Which one of the following drugs is both an opioid agonist and antagonist?

   A Aripiprazole

   B Naltrexone

   C Buprenorphine

   D Methadone

   E Gabapentin

94. Hallervorden–Spatz syndrome is a neurodegenerative disorder that results from lesions localizing to the:

   A Frontal lobes

   B Parietal lobes

   C Occipital lobes

   D Basal ganglia

   E Hippocampus

95. A patient presents to the emergency room because of alcohol withdrawal. He and his family describe a history of alcohol-induced blackouts. Which one of the following memory problems is most consistent with alcohol induced blackouts?

   A Making up details of how he got to work 3 days ago

   B Retrograde amnesia

   C Anterograde amnesia

   D Loss of memories from his daughter’s birthday 5 years ago

   E Inability to tell you who the current President is

96. Strict vegetarians who ingest no meat products can suffer from deficits in proprioception and vibration sensation due to lesions that localize to the:

   A Posterior spinal cord

   B Central spinal cord

   C Anterior spinal cord

   D Thalamus

   E Peripheral sensory nerves

97. A psychiatrist is treating a bipolar patient with carbamazepine. After being started on the drug he has therapeutic serum levels. Three months later the patient starts to become hypomanic and the psychiatrist decides to check a level. The level comes out below therapeutic range. Both the patient and his family reassure the psychiatrist that he has been taking the medication regularly. What should the psychiatrist do with this patient?

   A Confront him, because he and his family are lying

   B Stop the carbamazepine and put him on divalproex sodium

   C Increase the dose of carbamazepine and take follow-up serum levels

   D Add a high dose of a selective serotonin reuptake inhibitor

   E Hospitalize the patient

98. Patients exposed to isoniazid (INH) for tuberculosis treatment can develop a sensory polyneuropathy as a result of a deficiency in which one of the following?

   A Vitamin B6

   B Vitamin B12

   C Niacin

   D Thiamine

   E Vitamin A

99. What is the best way to handle suicidal patients with borderline personality disorder?

   A Take the threats seriously and take whatever steps are necessary to protect these patients

   B Do not discuss suicide with them

   C Isolate these patients from friends and family

   D Make these patients promise not to hurt themselves (contract for safety)

   E Give these patients benzodiazepine prescriptions to calm them down

100. The West Nile virus is considered to belong to which one of the following viral families?

   A Arenaviruses

   B Arboviruses

   C Filoviruses

   D Papovaviruses

   E Retroviruses

101. A patient is on lithium, risperidone, and a daily aspirin. He comes to his session confused and unsteady on his feet. He states that he has the flu, because of recent GI upset. Which one of the following should the psychiatrist do with this patient first?

   A Refer him to an internist

   B Get a lithium level

   C Review the patient’s recent diet

   D Send stool for ova and parasites

   E Obtain a complete blood count

102. Which one of the following neurotransmitters localizes predominantly to the basal forebrain and is responsible for memory, attention, and executive functioning?

   A Serotonin

   B Norepinephrine

   C GABA

   D Glycine

   E Acetylcholine

103. Which one of the following drugs does not act by blocking the reuptake of norepinephrine into the presynaptic neuron?

   A Imipramine

   B Venlafaxine

   C Bupropion

   D Nefazodone

   E Mirtazapine

104. What is the mechanism of action of the hallucinogen PCP (phencyclidine)?

   A Dopamine antagonism

   B Serotonin antagonism

   C Norepinephrine antagonism

   D NMDA antagonism

   E Acetylcholinesterase inhibition

105. A physician needs to give a benzodiazepine to someone with impaired liver function. Which one of the following would be the best choice of medication in this situation?

   A Diazepam

   B Oxazepam

   C Clonazepam

   D Prazepam

   E Estazolam

106. Which one of the following fungal organisms can cause vertebrobasilar strokes by invasion of vessel walls and tends to colonize in the paranasal sinuses and cause a hypersensitivity pneumonitis?

   A Histoplasma

   B Candida albicans

   C Aspergillus

   D Cryptococcus neoformans

   E Pseudallescheria boydii

107. Which one of the following side effects is most likely to develop in a patient started on fluoxetine?

   A Loss of consciousness

   B Shuffling gait

   C Headache

   D High blood pressure

   E Blurred vision

108. The GABA-A receptor (the most predominant GABA receptor) is which one of the following:

   A A sodium channel

   B A chloride channel

   C A calcium channel

   D A potassium channel

   E A magnesium channel

109. A psychiatrist made a minor error in her last session with a patient. The patient comes to her for psychodynamic psychotherapy. The best approach is to:

   A Interpret the patient’s reaction

   B Ignore the mistake

   C Give a long but clear explanation of her reasoning

   D Briefly acknowledge that she made a mistake

   E Profusely apologize

110. A failure to develop a cohesive self-awareness is known as:

   A Entrapment

   B Climactereium

   C Identity diffusion

   D Activity dependent modulation

   E All-or-none phenomenon

111. What is the most important step in treating separation-anxiety disorder in an 11-year-old?

   A Give methylphenidate

   B Give risperidone

   C Rapidly send the child back to school

   D Thorough psychoanalysis of the mother

   E High dose benzodiazepine treatment

112. Why do doctors use naltrexone for alcohol abuse?

   A It is almost 100% effective

   B It blocks the effects of alcohol at the GABA receptor

   C It alters dopamine levels to decrease pleasure from drinking

   D It has been shown to decrease craving and decrease alcohol consumption

   E It is better than behavioral modification in treating alcohol abuse

113. A 30-year-old man on imipramine complains of difficulty urinating and impotence. What should his doctor do for him?

   A Increase the dose of imipramine

   B Tell the patient to decrease fluid intake

   C Tell the patient to stop all sexual activity

   D Prescribe bethanechol

   E Prescribe melatonin

114. Which one of the following statements about mood disorders is true?

   A Major depression is more common in men than in women

   B Bipolar disorder has equal prevalence for men and women

   C Higher socioeconomic status leads to increased depression

   D There is a correlation between the hyposecretion of cortisol and depression

   E About 90% of those with major depressive disorder receive specific treatment

115. A mother brings a 26-month-old child into the doctor’s office. The child has not spoken any words yet. How should the doctor proceed?

   A Speech therapy

   B Audiometry

   C Sensory evoked potentials

   D Tell the mother to give it more time

   E Chromosomal analysis

116. The assisted recall of information by a person in the same external environment that the information was originally acquired in is known as:

   A Classical conditioning

   B Social learning

   C Partial recovery

   D Respondent conditioning

   E State-dependent learning

117. A 29-year-old man comes into the hospital with complaint of confusion, ataxia, disorientation and dysarthria. He has the smell of alcohol on his breath. Which one of the following is the best first step for the physician to take?

   A Phone the patient’s primary care physician

   B Speak with the patient’s family

   C Give intravenous thiamine

   D Sedate the patient with haloperidol

   E Give the patient an anticonvulsant

118. Niacin deficiency (pellagra) results in which one of the following classic triads of symptoms?

   A Gastritis, neuropathy, stroke

   B Dementia, dermatitis, diarrhea

   C Neuropathy, ataxia, dementia

   D Neuropathy, retinopathy, areflexia

   E Neuropathy, spasticity, encephalopathy

119. A psychiatrist is doing psychodynamic psychotherapy with a patient. The patient is usually on time, but missed a session last Tuesday. When he comes back, how should the psychiatrist approach this issue?

   A Do not mention the missed appointment

   B Refuse to treat the patient anymore

   C “You missed your appointment Tuesday. I was wondering what happened.”

   D “I’m glad you didn’t show on Tuesday. I spent the time with a patient I like better than you.”

   E “I’m going to charge you twice the normal fee because you missed your appointment last Tuesday.”

120. Which one of the following is not a sign of cannabis intoxication?

   A Conjunctival injection

   B Increased appetite

   C Dry mouth

   D Bradycardia

   E Orthostatic hypotension

121. A patient comes to his psychiatrist’s office with complaints consistent with akathisia. Which one of the following would be the best treatment?

   A Bupropion

   B Amoxapine

   C Vitamin B6

   D Captopril

   E Propranolol

122. What is the mechanism by which clonidine is effective in reducing symptoms of opiate withdrawal?

   A Indirect dopamine blockade

   B Serotonin increase in the locus ceruleus

   C Agonist activity at alpha-2 adrenergic receptors

   D Generation of the metabolite trichloroethanol

   E Decreased free T4

123. Blockade of muscarinic cholinergic receptors will not lead to which one of the following?

   A Difficulty urinating

   B Improvement in Alzheimer’s symptoms

   C Dry mouth

   D Blurred vision

   E Delirium

124. Which one of the following lab tests is most likely to pick up alcohol abuse?

   A Gamma glutamyl transferase (GGT)

   B Mean corpuscular volume (MCV)

   C Uric acid

   D Serum glutamic-oxaloacetic transaminase (SGOT)

   E Serum glutamic-pyruvic transaminase (SGPT)

125. A psychiatrist is asked by a primary care physician to treat a patient with Tourette’s disorder. He is suffering from several motor tics. Which one of the following would be the best medication to give him?

   A Propranolol

   B Pimozide

   C Paroxetine

   D Pindolol

   E Piroxicam

126. On which chromosome is the gene for amyloid precursor protein found?

   A Chromosome 19

   B Chromosome 20

   C Chromosome 21

   D Chromosome 4

   E Chromosome 13

127. Which one of the following answer choices is central to Kohut’s theories of self psychology?

   A The theory of oedipal conflict

   B The concept of the good enough mother

   C The paranoid–schizoid position

   D The necessity for parental mirroring and empathic responsiveness to the child

   E The importance of the depressive position

128. A patient with bipolar disorder is on carbamazepine. He goes to his primary care physician and is placed on erythromycin. What should be expected to happen?

   A Carbamazepine levels will go down

   B Erythromycin levels will go down

   C No interaction of any kind

   D Carbamazepine levels will go up

   E Erythromycin levels will go up

129. A patient with bipolar disorder gives birth to a child with spina bifida and hypospadias. What is the most likely cause for the child’s defects?

   A Genetics

   B Intrauterine infection

   C Haloperidol use during pregnancy

   D Valproic acid use during pregnancy

   E Lithium use during pregnancy

130. A patient presents with decreased energy, increased appetite, weight gain, increased sleep, decreased mood, lack of interest in usual activities, and social withdrawal. Which one of the following medications would be the best choice to treat him?

   A Citalopram

   B Lithium

   C Desipramine

   D Phenelzine

   E Venlafaxine

131. A patient states that he was given an antidepressant in the past but does not remember the name. He does remember having his blood pressure checked regularly by the psychiatrist because of the antidepressant. Which one of the following was the patient most likely taking?

   A Mirtazapine

   B Paroxetine

   C Venlafaxine

   D Citalopram

   E Fluoxetine

132. Which one of the following is the focus of interpersonal therapy?

   A Anxiety management

   B Belief systems

   C Faulty cognitions

   D Social interactions

   E Transference

133. Which one of the following is not an aspect of experiments carried out by Nikolas Tinbergen?

   A Quantifying the power of certain stimuli in eliciting specific behavior

   B Displacement activities

   C Innate releasing mechanisms

   D Autism

   E Imprinting

134. Which one of the following structures is most critical to the formation of memory?

   A Right frontal lobe

   B Right parietal lobe

   C Thalamus

   D Cerebellum

   E Hippocampus

135. Which one of the following is not a rating scale used for mood disorders?

   A Beck depression inventory

   B Zung self-rating scale

   C Carroll rating scale

   D Montgomery–Åsberg scale

   E Brief psychiatric rating scale (BPRS)

136. Which one of the following is not a characteristic of sleep terror disorder?

   A Awakening from sleep and screaming

   B Autonomic arousal

   C Recall of a detailed dream

   D Sweating

   E Unresponsiveness to attempts to comfort the person during the episode

137. Thioridazine is most often associated with which one of the following side effects?

   A Hematuria

   B Delayed orgasm

   C Retrograde ejaculation

   D Priapism

   E Hypospadias

138. What is the mechanism of action of donepezil?

   A Dopamine blockade

   B Serotonin reuptake inhibition

   C Acetylcholinesterase inhibition

   D Increasing GABA activity

   E Prevention of beta amyloid deposition

139. Depression may present differently in different cultures. How would the knowledgeable psychiatrist predict depression would present in a 37-year-old Chinese immigrant?

   A Concern with mood symptoms

   B Somatic complaints

   C Hysteria

   D Self mutilation

   E Paranoia

140. Which one of the following neurotransmitters is most involved in the effects of methylenedioxyamphetamine (ecstasy)?

   A Serotonin

   B Norepinephrine

   C GABA

   D Glycine

   E Acetylcholine

141. The growth of child guidance clinics in the US in the early 1900s lead to:

   A The development of the first medications for ADHD

   B The development of sewage systems in major US cities

   C The development of child psychiatry as a profession

   D Freud’s three essays on the theories of sexuality

   E The Ryan White Care Act

142. The most common neurologic manifestation of neurosarcoidosis is:

   A Cranial neuropathy

   B Cauda equina syndrome

   C Peripheral neuropathy

   D Meningoencephalitis

   E Uveitis

143. Trazodone is most often associated with which one of the following side effects?

   A Hematuria

   B Delayed orgasm

   C Retrograde ejaculation

   D Priapism

   E Hypospadias

144. Which one of the following answer choices is true regarding aggression?

   A High levels of cerebrospinal fluid (CSF) serotonin are associated with increased aggression

   B Serotonin is unrelated to aggression

   C Low levels of CSF serotonin are associated with increased aggression

   D Low levels of CSF serotonin are associated with decreased aggression

   E Low levels of dopamine are associated with increased aggression

145. How does mirtazapine work?

   A Serotonin reuptake inhibition

   B Norepinephrine reuptake inhibition

   C Alpha-2 adrenergic receptor antagonism

   D Partial dopamine antagonism

   E Decreasing breakdown of serotonin in the synaptic cleft

146. Which one of the following answer choices is most consistent with sleep changes in the elderly?

   A Increased REM sleep only

   B Increased slow wave sleep only

   C Increased REM and slow wave sleep

   D Decreased REM and slow wave sleep

   E Decreased slow wave sleep only

147. A score of 70 on the Global Assessment of Functioning corresponds with:

   A Persistent failure to maintain personal hygiene

   B Major impairment in several areas

   C Superior functioning in all areas

   D Some difficulty functioning, but generally functioning well

   E No friends, unable to keep a job

148. Which one of the following is not an aspect of Kleine–Levin syndrome?

   A Irritability

   B Voracious eating

   C Loss of libido

   D Incoherent speech

   E Hypersomnia

149. David works 17 hours per day. He does not have many friends because he feels that they interfere with his work schedule. He believes that he is a moral person and harshly criticizes those whom he finds to be unethical. He often starts projects but fails to complete them because he can not do them perfectly. His family describes him as stubborn and cheap, because he will never throw anything out. David’s most likely diagnosis is:

   A Generalized anxiety disorder

   B Obsessive–compulsive disorder

   C Obsessive–compulsive personality disorder

   D Schizoid personality disorder

   E Avoidant personality disorder

150. Which one of the following is an important technique of cognitive behavioral therapy?

   A Maintaining therapeutic neutrality

   B Offering interpretations of patients’ unconscious wishes

   C Abreaction

   D Working through unresolved conflict

   E Finding and testing automatic thoughts

Answer Key – Test Number Four

1. C

2. A

3. A

4. D

5. B

6. C

7. A

8. D

9. E

10. B

11. A

12. A

13. E

14. D

15. D

16. E

17. D

18. B

19. D

20. D

21. D

22. A

23. B

24. B

25. E

26. E

27. E

28. A

29. D

30. C

31. E

32. E

33. D

34. D

35. E

36. A

37. C

38. B

39. E

40. D

41. D

42. B

43. A

44. A

45. C

46. B

47. C

48. C

49. D

50. A

51. D

52. B

53. D

54. D

55. E

56. A

57. D

58. E

59. D

60. D

61. D

62. B

63. D

64. C

65. E

66. D

67. D

68. A

69. B

70. D

71. B

72. C

73. E

74. B

75. D

76. A

77. C

78. D

79. A

80. B

81. E

82. C

83. C

84. B

85. E

86. C

87. A

88. A

89. D

90. E

91. D

92. B

93. C

94. D

95. C

96. A

97. C

98. A

99. A

100. B

101. B

102. E

103. E

104. D

105. B

106. C

107. C

108. B

109. D

110. C

111. C

112. D

113. D

114. B

115. B

116. E

117. C

118. B

119. C

120. D

121. E

122. C

123. B

124. A

125. B

126. C

127. D

128. D

129. D

130. D

131. C

132. D

133. E

134. E

135. E

136. C

137. C

138. C

139. B

140. A

141. C

142. A

143. D

144. C

145. C

146. D

147. D

148. C

149. C

150. E

Explanations – Test Number Four

Question 1. C. Freud described a series of stages through which children pass as a part of normal development. These stages correspond to shifts of sexual energy from one erotic body part to the next.

    The first phase is the oral phase from birth to 1 year. In the oral phase the infant’s needs and expression are centered in the mouth, lips, and tongue. Tension is relieved through oral gratification. Those who do not complete the oral phase successfully can be very dependent in adulthood. Successful resolution of the oral stage allows the adult to both give and receive without excessive dependency or envy.

    The next is the anal stage from ages 1 to 3 years. In the anal stage the child develops control of his anal sphincter. This phase is marked by increase in aggressive and libidinal drives. Control of feces gives the child independence, and the child struggles with the parent over separation. Successful completion of this stage leads to a sense of independence from the parent. Failure to complete this stage leads to obsessive–compulsive neuroses.

    The phallic stage is from ages 3 to 5 years. In the phallic stage there is a focus on sexual interests and excitement in the genital area. The goal of this stage is to focus erotic interests in the genital area and lay the groundwork for gender identity. Poor resolution of this stage leads to the neuroses often associated with poor resolution of the oedipal complex. Successful resolution leads to a clear sense of sexual identity, curiosity without embarrassment, initiative without guilt, and mastery over things both internal and external.

    The phallic stage is followed by latency, during which there is a decrease in sexual interest and energy. Latency lasts from age 5 years until puberty. It is a period of consolidating and integrating previous development in psychosexual functioning and developing adaptive patterns of functioning.

    At puberty there is an increase in sexual energy. This time is described as the genital stage. It lasts from about ages 11–13 years until adulthood. In this stage, libidinal drives are intensified. There is a regression in personality organization, allowing for resolution of prior conflicts and the solidification of the adult personality. The goal is the ultimate separation from the parents and the development of non-incestuous object relations. Failure to complete this stage can lead to multiple complex outcomes. Freud felt that one must pass successfully through all of these stages to develop normal functioning as an adult.

    Human Development

    K&S Ch. 2

Question 2. A. Amaurosis fugax is a symptom of carotid artery territory ischemia. It presents as a sudden onset of transient loss of vision that manifests as a curtain or shade or veil usually over the central visual field. The duration of visual loss is generally brief, lasting about one to five minutes, and only infrequently exceeds thirty minutes in duration. When the episode concludes, vision is generally returned to normal. The event is then truly termed a transient ischemic attack (TIA), because of a duration less than twenty-four hours. In some cases, there is permanent visual loss due to retinal infarction. Amaurosis fugax is the only feature that can distinguish a middle cerebral artery syndrome from a carotid artery syndrome.

    Vertebrobasilar territory ischemia would be expected to affect the cerebellum and/or brainstem. Classic posterior circulation ischemic symptoms include ataxia, nystagmus, vertigo, dysarthria, and dysphagia. An ipsilateral Horner’s syndrome can occur if the descending oculosympathetic fibers are disrupted. Crossed weakness (ipsilateral facial paresis, contralateral limb paresis) is indicative of brainstem involvement above the level of the area of decussation of the pyramids in the medulla.

    Lenticulostriate territory ischemia affects the small penetrating branching arteries off of the middle cerebral artery that feed the striatum. Ischemia to this territory can produce lacunar infarcts of the internal capsule that result in a pure contralateral motor hemiparesis.

    Anterior cerebral artery ischemia would be expected to produce a contralateral hemiparesis of the leg preferentially, because the cortical homuncular representation of the leg is situated parasagittally in the postcentral motor cortex.

    Middle cerebral artery territory ischemia can take many different forms. If the lesion is in the dominant hemisphere, aphasia may result. Nondominant ischemia can result in hemineglect, anosognosia, visual and tactile extinction, aprosody of speech and contralateral limb apraxia.

    Neurology

    B&D Ch. 51

Question 3. A. Norepinephrine (NE) is made in the locus caeruleus. Serotonin is made in the dorsal raphe nuclei. Dopamine is made in the substantia nigra. Acetylcholine is made in the nucleus basalis of Meynert.

    Basic Neuroscience

    K&S Ch. 3

Question 4. D. The onset of back pain, followed by leg weakness and urinary incontinence is a classic manifestation of bilateral spinal cord pathology. In this case, the correct syndrome is a transverse myelitis because the deficits are closely preceded by an influenza vaccination. Transverse myelitis is a segmental inflammatory syndrome of the bilateral spinal cord. It is believed to be immunologic in origin and often follows an infection or vaccination, or is the direct result of demyelination due to multiple sclerosis (MS). Up to 40% of cases have no identifiable origin. The classic presentation is the rapid onset of bilateral leg weakness that presents with a clear-cut sensory level below the level of the lesion. Pain and temperature sensation are usually affected, but often proprioception and vibration sensation are spared. Urinary and/or bowel incontinence are common findings. MRI of the spine is the imaging modality of choice. If both the spine and the brain show demyelination on MRI imaging the likelihood of the myelitis being the first manifestation of MS is greater than 50%. If the condition is proven to be a result of inflammation or demyelination, then high dose intravenous steroids are the initial acute treatment of choice.

    Spinal cord metastases causing an acute cord compression would be expected to cause a similar presentation to myelitis as noted above, but one would expect the history to reveal some sort of primary cancer, such as that of the lung or prostate, that would precede the onset of spinal cord symptoms. Only about 15% of primary CNS tumors originate in the spinal cord. Emergent MRI must be performed to characterize the lesion. Treatment of acute cord compression due to metastatic lesions involves intravenous steroids, radiation therapy, and possible surgical decompression.

    Vacuolar myelopathy is found almost uniquely in AIDS patients, and is similar to the condition noted in vitamin B12 deficiency. AIDS myelopathy is noted in about one-quarter of AIDS patients. The clinical picture is similar to that of myelitis or cord compression, but the timeline is usually much slower and more progressive, evolving over many months. The pathophysiology of AIDS-related vacuolar myelopathy may be due to viral release of cytokines that are neurotoxic in nature, or abnormalities in vitamin B12 utilization. Vitamin B12 levels are often normal in these cases. Antiretroviral therapy may not reverse the symptoms.

    Acute disseminated encephalomyelitis (ADEM) is a monophasic demyelinating syndrome that follows a systemic infection or vaccination. It differs from transverse myelitis, because ADEM involves the whole CNS and is not simply localized to a segment of the spinal cord. ADEM usually results in multifocal signs and symptoms that include an encephalopathy. Brainstem and cerebral involvement are noted, as well as symptoms localizing to the spinal cord.

    Neurology

    B&D Chs 55&56

Question 5. B. If different doctors can look at the same case and make the same diagnosis, the diagnosis is said to be reliable. In other words, it is consistent. Accuracy of the diagnosis is called validity. Descriptive validity means that the disorder has features that are characteristic enough to separate it from other disorders. Predictive validity means that the diagnosis allows the doctor to predict clinical course and treatment response. Construct validity means the diagnosis is based on underlying pathophysiology and the use of biologic markers to confirm the disease.

    Statistics

    K&S Ch. 4

Question 6. C. Horner’s syndrome results from an interruption to the sympathetic fibers supplying the pupil, upper eyelid, facial sweat glands and facial blood vessels. The classic symptom triad is that of ptosis, miosis, and anhydrosis. Horner’s syndrome can be seen as part of the lateral medullary stroke syndrome (Wallenberg’s syndrome), carotid occlusion or dissection, high spinal cord lesions, neoplasms like Pancoast’s tumor that affect the cervical ganglia, or intracranial hemorrhage. If the condition affects only the eye and not the sweat glands, the lesion usually localizes to the territory of the internal carotid artery. The other answers to this question need no explanation as they are simply nonsense distracters.

    Neurology

    B&D Ch. 77

Question 7. A. This question lists side effects that may be found with amphetamines. All of the answer choices except choice A, are central nervous system depressants and would be predicted to have effects opposite to many of those listed in the question, although there may be some similarities with certain symptoms. Amphetamine is a sympathomimetic.

    Substance Abuse and Addictive Disorders

    K&S Ch. 12

Question 8. D. Prosopagnosia is the inability to recognize familiar faces. It is associated classically with bilateral occipital–temporal lesions. It is often associated with agraphia and achromatopsia (inability to recognize colors and hues). It is almost always associated with visual field deficits. Anosognosia, the denial of disease or hemiparesis, is seen with lesions of the non-dominant hemisphere. Simultanagnosia is the inability to perceive a scene with multiple parts to it. The patient is only able to see and recognize individual elements of a multi-part scene, and cannot interpret the overall picture. Astereognosis refers to the inability to recognize and identify items by weight, texture and form alone, when the items are held in the hand. It is a form of tactile agnosia. Aprosodia is a deficit in the emotional aspect of expressive or receptive speech.

    Neurology

    B&D Chs 11,12&51

Question 9. E. With PCP use hypertension is often seen, not hypotension. Other symptoms include paranoia, nystagmus, catatonia, convulsions, hallucinations, mood lability, loosening of associations, violence, mydriasis, ataxia, and tachycardia. Treatment consists of haloperidol every 2–4 hours until the patient is calm. PCP can be detected in the urine up to eight days after ingestion.

    Substance Abuse and Addictive Disorders

    K&S Ch. 12

Question 10. B. Bell’s palsy (VII nerve palsy) is a severe, acute, unilateral, complete facial paresis that evolves over 24–48 hours. The condition is often accompanied by pain behind the ear. Taste impairment and hyperacusis are frequent associated symptoms. The incidence is about 20 in 100 000. The syndrome is believed to be of viral etiology, and herpes simplex virus is thought to be the most frequent viral pathogen responsible for the condition. About 80–85% of patients improve completely within three months. Incomplete paralysis at onset is a better prognostic sign than complete paralysis. Treatment with acyclovir and prednisone is often administered, but remains controversial due to the etiological uncertainty of the disorder. The paralysis of Bell’s palsy involves the entire face, whereas the facial paresis of a hemispheric stroke spares the upper third of the face (the brow and upper eyelid).

    Neurology

    B&D Ch. 70

Question 11. A. This question focuses on Piaget’s stages of cognitive development. Object permanence develops during the sensorimotor stage. There are several other questions on Piaget in this book. If you did not get this question correct, go back and review those questions and answer explanations. Know Piaget well!

    Human Development

    K&S Ch. 2

Question 12. A. Syringomyelia refers to the constellation of signs and symptoms produced by a syrinx, a cavitation of the central spinal cord. The cavitation may be contiguous with a dilated central spinal canal, or it may be separate from a central canal. Most syringes occur in the cervical spinal cord. Most of those that develop from a central spinal canal are associated with an Arnold–Chiari type I or type II malformation. The classic presentation is that of a dissociated suspended sensory deficit usually in a cape or shawl pattern over the arms and upper trunk. There is impairment of pain and temperature sensation and preserved light touch, vibration and proprioception. These sensory deficits are combined with lower motor neuron signs (flaccidity, muscular atrophy, fasciculations) at or about the level of the lesion, as well as spinal long tract signs below the level of the lesion. Pain often accompanies a syrinx and can manifest as headache, neck pain, radicular pain, and segmental dysesthesia. MRI of the spine is the imaging modality of choice to best evaluate a syrinx.

    Neurology

    B&D Ch. 73

Question 13. E. Urinary retention is a side effect of the belladonna alkaloids, but not of glue and benzene products. All of the other answer choices are possibilities for a patient sniffing glue daily for over 6 months. In addition to those contained in the question, side effects include slurred speech, ataxia, hallucinations, and tachycardia with ventricular fibrillation. This grouping of substances includes glue, benzene, gasoline, paint thinner, lighter fluid, and aerosols.

    Substance Abuse and Addictive Disorders

    K&S Ch. 12

Question 14. D. Opsoclonus–myoclonus is a paraneoplastic movement disorder that is also called “dancing eyes–dancing feet” syndrome. It is seen most often in infants ages 6 to 18 months and in 50% or more of cases is associated with an infantile neuroblastoma. In adults, lung cancer is a common cause of this disorder. It can also be noted as a result of postviral encephalitis, multiple sclerosis, thalamic hemorrhage, and hyperosmolar coma. The condition presents as multifocal myoclonus and rapid dancing movements of the eyes. It is caused by lesions to the pause cells in the pons. Steroids or adrenocorticotropic hormone are effective treatment for this particular type of myoclonus. The other conditions noted in this question are distracters that are not related to opsoclonus–myoclonus.

    Neurology

    B&D Ch. 52G

Question 15. D. Social phobia is characterized by a fear of one or more social or performance situations in which the person is exposed to unfamiliar people or possible scrutiny by others. The individual is afraid of acting in a way that would be embarrassing. Exposure to the situation almost always causes anxiety, and the person is aware that the fear is excessive. In this question, fear of scrutiny is the most definitive symptom of social phobia. The other choices could provoke anxiety in someone with some type of anxiety disorder but are neither necessarily limited to nor considered major diagnostic criteria for social phobia.

    Anxiety Disorders

    K&S Ch. 16

Question 16. E. The Ramsay Hunt syndrome is a herpetic cranial neuritis that affects the facial (VII) and acoustic (VIII) nerves. The pathogen is of course the varicella-zoster virus (herpes zoster; chicken pox virus). The clinical presentation is that of a painful facial palsy, vertigo, ipsilateral hearing loss and vesicles in the external auditory canal and sometimes on the pinna. The infection is presumed to localize to the geniculate ganglion. Treatment is symptomatic. Recovery is often worse than in idiopathic Bell’s palsy. The other answer choices are distracters.

    Neurology

    B&D Chs 53&69

Question 17. D. All of the answer choices given are true except for D. In cases of body dysmorphic disorder, medical, dental, or surgical treatment fails to solve the patient’s preoccupation. The disorder consists of a preoccupation with an imagined defect in appearance. If a slight physical anomaly is present the concern is markedly excessive. The cause of this disorder is unknown, onset is usually slow, and marked impairment in functioning occurs. There is an association with mood disorders and in some studies as many as 50% of patients benefited from treatment with a serotonin-selective reuptake inhibitor. The body part of concern may change over the course of the disorder. Differential diagnosis should include anorexia, gender identity disorder, and conditions that can result in brain damage such as neglect syndromes.

    Somatic Symptom Disorders

    K&S Ch. 17

Question 18. B. Glioblastoma multiforme is the most common primary brain tumor in about 50% or more of patients over 60 years of age. Average survival is about one year after diagnosis with radiation therapy. Anaplastic astrocytoma has a bimodal peak in incidence in the first and third decades. Ependymomas make up about 5% of all brain tumors. They are the third most common CNS tumors in children. Meningiomas make up about 20–25% of all brain tumors. They are more likely to occur after 50 years of age, and occur about twice as frequently in females as in males. About 80% of meningiomas turn out to be benign. Acoustic neuromas are considered to be schwannomas (nerve sheath tumors). The peak incidence of acoustic neuromas is in the fourth and fifth decades.

    Neurology

    B&D Ch. 52

Question 19. D. Neuroimaging studies in obsessive–compulsive disorder (OCD) patients have demonstrated abnormalities in the caudate, thalamus, and orbitofrontal cortex. Functional neuroimaging by positron emission tomography (PET) scanning has demonstrated increased metabolism in the basal ganglia (predominantly in the caudate), the frontal lobes and the cingulate gyrus. CT and MRI of the brains of patients with OCD have demonstrated bilaterally smaller caudate nuclei than those seen in normal controls.

    Laboratory Tests in Psychiatry

    K&S Ch. 16

Question 20. D. Anton’s syndrome is considered to be an agnosia characterized by cortical blindness. The lesion localizes to the bilateral occipital lobes, usually due to strokes, particularly in the calcarine cortex (Brodmann’s area 17) and visual association cortex. The hallmark of the syndrome is that patients deny that they are blind and they confabulate. Patients may also suffer from visual hallucinations.

    Neurology

    B&D Ch. 14

Question 21. D. Signs of impending violence, or predictive factors for violence include alcohol and drug intoxication, recent acts of violence, command auditory hallucinations, paranoia, menacing behavior, psychomotor agitation, carrying weapons, frontal lobe disease, catatonic excitement, certain manic or agitated depressive episodes, violent ideation, male gender, ages 15–24 years, low socioeconomic status, and few social supports.

    Management in Psychiatry

    K&S Ch. 34

Question 22. A. Riluzole (Rilutek) is a glutamate antagonist and is FDA-approved in the treatment of amyotrophic lateral sclerosis (ALS). It has demonstrated mild to moderate improvement in the survival rate of ALS. It may also help patients remain in a milder disease state for a longer period of time. The medication comes in 50 mg tablets which are started at one per day at night and increased to twice daily after 1–2 weeks, if tolerated. Side effects include gastrointestinal upset, dizziness, fatigue, and liver enzyme elevation. The other answer choices are distracters.

    Neurology

    B&D Ch. 74

Question 23. B. Conversion disorder consists of one or more neurological symptoms that can not be explained by a medical condition. The symptoms are thought to be unconsciously produced in response to psychological conflict. Pseudoseizures can be common symptoms of conversion disorder. Conversion disorder can be associated with passive–aggressive, antisocial, histrionic, and dependent personality disorders. In obsessive–compulsive disorder a patient has intrusive unwanted thoughts that cause them to repeat a ritual or action to remove the anxiety associated with that thought. Nothing like that is described in this question. Somatization disorder presents as a patient who has a number of medical complaints involving several organ systems that can not be otherwise explained by a known medical condition. That is not the case in this question. Social phobia presents as a patient who has a fear of being in social situations or meeting new people. That has nothing to do with this question. Panic disorder presents as a patient who has recurrent and unexpected panic attacks. This patient is having pseudoseizures, not panic attacks.

    Somatic Symptom Disorders

    K&S Chs 16&17

Question 24. B. Primary central nervous system lymphoma occurs in AIDS patients in about 5% of cases. These lymphomas are most often of B-cell origin. Patients can present with a gradual onset of any of the following symptoms: headache, aphasia, hemiparesis, altered mental status, behavioral changes, and ataxia. Constitutional symptoms like fever and weight loss are generally absent. Diagnosis is established by MRI brain imaging. PCR testing on CSF revealing Epstein–Barr virus DNA helps to corroborate the diagnosis. Treatment with antiretroviral polypharmacy can help slow progression. None of the other tumor types mentioned in the answer choices are particularly associated with AIDS.

    Neurology

    B&D Ch. 53

Question 25. E. The development of the central nervous system is a very important area of study, as abnormal development is implicated in several clinical conditions, including schizophrenia. The central and peripheral nervous systems arise from the neural tube. The neural tube gives rise to the ectoderm, which becomes the peripheral nervous system, whereas the neural tube itself becomes the central nervous system. The second trimester of gestation is the peak of neuronal proliferation, with 250 000 neurons born each minute. Migration of neurons, guided by glial cells, peaks during the first 6 months of gestation. Synapse formation occurs at a high rate from the second trimester through age 10, but peaks around 2 years (toddler period) with as many as 30 million synapses forming per second. The nervous system is also actively myelinating its axons starting prenatally, and continuing through childhood, finishing in the third decade of life.

    Basic Neuroscience

    K&S Ch. 3

Question 26. E. AIDS dementia complex is a late complication of AIDS, particularly when the HIV infection is untreated and CD4 cell counts are low. Symptoms include poor attention and concentration, bradyphrenia, forgetfulness, poor balance, uncoordination, personality changes, apathy, and depression. Focal neurologic deficits are usually absent, such as hemiparesis or aphasia. Treatment involves administration of antiretroviral therapy.

    Neurology

    B&D Ch. 53

Question 27. E. All of the answer choices given are correct, with the exception of increased bone density. Anorexic patients tend to suffer from osteoporosis. They also have cachexia, loss of muscle mass, reduced thyroid metabolism, loss of cardiac muscle, arrhythmias, delayed gastric emptying, bloating, abdominal pain, amenorrhea, lanugo (fine baby-like hair), and abnormal taste sensation.

    Feeding and Eating Disorders

    K&S Ch. 23

Question 28. A. Central nervous system toxoplasmosis is the most frequent CNS opportunistic infection in the AIDS patient. It is noted in 10% or more of AIDS patients. The infection is caused by the parasite Toxoplasma gondii. Cerebral toxoplasmosis in the AIDS patient results most often from a resurgence of a previously acquired infection. The infection usually occurs in late-stage AIDS, when CD4 cell counts are less than 200/μL. CNS toxoplasmosis presents clinically with headache and focal neurological deficits with or without fever. Other possible manifestations include aphasia, seizures, and hemiparesis. In cases with significant progression, patients can develop confusion and lethargy that lead to coma. The diagnosis is made on the basis of CT or MRI brain imaging revealing a single lesion or multiple ring-enhancing lesions. Therapy is undertaken with pyrimethamine and sulfadiazine, or with clindamycin in sulfa-allergic patients. The other answer choices occur less frequently in AIDS patients. Cryptococcal meningitis, another important opportunistic infection in AIDS, as well as the other three answer choices, are explained elsewhere in this volume.

    Neurology

    B&D Ch. 53

Question 29. D. All choices given are correct, except for hyperkalemia. In purging, it is common to see a hypokalemic, hyperchloremic alkalosis. In addition, one may also see hypomagnesemia, pancreatic inflammation, increased serum amylase, esophageal erosion, and bowel dysfunction.

    Feeding and Eating Disorders

    K&S Ch. 23

Question 30. C. HIV-associated vacuolar myelopathy is the most common cause of spinal cord pathology in AIDS patients, and is seen in one-quarter to one-half of patients on autopsy. The disorder usually occurs in late-stage AIDS. The clinical picture is that of spasticity, gait instability, lower extremity weakness, loss of proprioception and vibration sensation, and sphincter dysfunction. Neurologic examination reveals a spastic paraparesis, hyper-reflexia and Babinski’s signs. It is unusual to note a clear-cut sensory level on the trunk. Vitamin B12 levels are usually normal. Viral neurotoxic cytokines may contribute to the pathophysiology of the disease.

    Distal sensory polyneuropathy is the most common peripheral nerve syndrome that complicates AIDS. It is seen in about one-third of AIDS patients. The clinical presentation is that of diminished ankle jerk reflexes, decreased pain, temperature, and vibration sensation, and possible paresthesia or numbness of the feet. The disorder is usually symmetrical. The condition frequently presents with lower extremity neuropathic pain. Treatment involves antiretroviral therapy and management of the pain with tricyclic antidepressants, anticonvulsants, or even narcotic analgesics if needed.

    Neurosyphilis, progressive multifocal leukoencephalopathy, and HTLV-1 myelopathy are all explained elsewhere in this volume.

    Neurology

    B&D Ch. 53

Question 31. E. Central serous chorioretinopathy is a disease leading to detachment of the retina and has nothing to do with anxiety. Carcinoid syndrome can mimic anxiety disorders and is accompanied by hypertension, and elevated urinary 5-HIAA. Hyperthyroidism presents with anxiety in the context of elevated T3, T4 and exophthalmos. Hypoglycemia presents with anxiety and fasting blood sugar under 50 mg/dL. Signs and symptoms of diabetes may also be present with hypoglycemia (polyuria, polydipsia, and polyphagia). Hyperventilation syndrome presents with a history of rapid deep respirations, circumoral pallor, and anxiety. It responds well to breathing into a paper bag.

    Anxiety Disorders

    K&S Ch. 16

Question 32. E. 

image

Progressive multifocal leukoencephalopathy (PML) is a demyelinating disorder affecting AIDS patients with low CD4 counts. It results from an opportunistic infection by the JC virus, a form of human papilloma virus. It occurs in about 5% of patients with AIDS. Demyelination occurs preferentially in the subcortical white matter of the parietal and/or occipital lobes. Clinical presentation can involve hemiparesis, aphasia, sensory deficits, ataxia, and visual field deficits. Mental status may deteriorate progressively over time. MRI often reveals multiple or coalesced nonenhancing white matter lesions in the parietal or occipital lobes. CSF assay for the JC virus DNA by PCR can help confirm the diagnosis. There is no specific treatment for PML and mean survival is about two to four months after diagnosis. Antiretroviral therapy can reconstitute immune function, but often proves to be too little too late. The other answer choices in this question are distracters and are explained elsewhere in this volume.

    Neurology

    B&D Ch. 53

Question 33. D. The distinction between social phobia and avoidant personality disorder can sometimes be confused. In a situation where the patient is afraid of almost all social situations, then avoidant personality disorder should be considered. Social phobia presents with a fear of one or more social situations. Avoidant personality disorder is defined by a pervasive pattern of social inhibition, feelings of inadequacy, and hypersensitivity to negative evaluation. This can be shown by avoiding activities involving interpersonal contact, unwillingness to get involved with people unless certain of being liked, showing fear in intimate relationships for fear of ridicule, preoccupation with criticism or rejection in social situations, viewing self as socially inept and being inhibited in new social situations or afraid to take risks for fear of embarrassment.

    Anxiety Disorders

    Personality Disorders

    K&S Ch. 27

Question 34. D. Prion diseases are also known as transmissible spongiform encephalopathies. Prions are proteinaceous and infectious particles that are unlike bacteria or viruses. Prions are unique in that they can be passed on through heredity (chromosome 20), acquired by infection, or acquired spontaneously. Kuru is the prion disease endemic to the cannibalistic Fore people. The clinical presentation is that of progressive cerebellar ataxia. The progressive disease course generally leads to death in about twelve months from initial onset.

    Gerstmann–Sträussler–Scheinker syndrome is an inherited form of prion disease. Symptoms begin to manifest in about the third or fourth decade. The disease is the slowest of the spongiform encephalopathies and can progress over several years. The clinical presentation depends on which mutation in the PRNP gene is acquired by the patient. The syndrome varies in nature and presents with any combination of ataxia, Parkinson’s plus-like symptoms, or a progressive dementia.

    Fatal familial insomnia occurs between ages 35 and 60 years. It presents with progressive insomnia and sympathetic autonomic hyperactivity, such as hyperhidrosis, tachycardia, hyperthermia, or hypertension. Mild cognitive impairment is usually noted. Other symptoms can include ataxia, tremor, myoclonus, confusion, or hallucinations.

    Creutzfeldt–Jakob disease can be sporadic, iatrogenic or familial. CSF evaluation of 14-3-3 and Tau proteins is the diagnostic test of choice to confirm the disease. The classic sporadic disease usually affects those between ages 50 and 75 years. The downward clinical course is rapid and progresses towards death over 6 months to 2 years. The disease has three stages. Stage one presents with neuropsychiatric symptoms that can include fatigue, sleep disturbance, memory and concentration deficits, and personality changes. Stage two presents with generalized cognitive deficits, as well as significant psychiatric symptoms such as psychosis and hallucinosis. The third and final stage that precedes death is characterized by profound and severe dementia, with myoclonus and choreoathetosis. Treatment of the spongiform encephalopathies is palliative and symptomatic. There is no known cure for these diseases.

    Devic’s disease, also known as neuromyelitis optica, is believed to be a variant of multiple sclerosis (MS). The presentation is that of cervical myelopathy and bilateral optic neuropathy. In certain patients, the two conditions occur simultaneously, but in others, they occur at separate intervals, pointing more directly to a diagnosis of MS. Devic’s disease is a disease of demyelination and has nothing to do with prions.

    Neurology

    B&D Chs 53&54

Question 35. E. Bipolar II disorder is characterized by at least one major depressive episode and at least one hypomanic episode during the patient’s lifetime. There are no full manic episodes in bipolar II disorder. If criteria for a manic episode are met then the correct diagnosis is bipolar I disorder. Psychotic features can be found in bipolar I disorder during mania or depression but in bipolar II will occur only associated with depression as full mania is not present in bipolar II.

    Bipolar Disorders

    K&S Ch. 15

Question 36. A. Carotid artery occlusion presents with the triad of headache, ipsilateral Horner’s syndrome, and contralateral hemiparesis. A resultant cerebral infarct can increase intracranial pressure and thereby cause headache. The Horner’s syndrome itself can result in headache because of interruption of ascending sympathetic tracts from the superior cervical ganglion to the intracranial vessels and dura which are pain-sensitive structures.

    Pontine hemorrhage is a neurologic emergency as it can lead to rapid death if the bleed is large. This hemorrhage type accounts for about 5% of all intracranial hemorrhages. Large bleeds in the basal tegmentum of the pons result in a clinical presentation characterized by quadriplegia, coma, decerebrate posturing, respiratory rhythm anomalies (apneustic breathing), pinpoint reactive pupils, hyperthermia, horizontal ophthalmoplegia, and ocular bobbing. The likely etiology of these bleeds is the rupture of small tegmental pontine penetrating arteries that originate from the trunk of the basilar artery.

    Cerebral aneurysmal rupture results in a sudden explosive headache (“the worst headache of my life”), obtundation, nausea, vomiting, meningeal signs, neck pain, and photophobia. Aneurysms are discussed in further detail elsewhere in this volume, as are giant cell arteritis and Wallenberg’s syndrome.

    Neurology

    B&D Chs 51&69

Question 37. C. Analysis of variance is a set of statistical procedures which compares two groups and determines if the differences are due to experimental influence or chance. Other answer choices were covered in Test Three.

    Statistics

    K&S Ch. 4

Question 38. B. The condition described in this question is of course that of a Bell’s palsy (VII nerve palsy). Bell’s palsy is characterized by a sudden, severe, unilateral infranuclear (i.e., involving both the upper and lower parts of the face) facial paresis. In serious cases, the brow droops, with widening of the palpebral fissure, and the eyelid cannot close completely. When the patient tries to close the affected eye with effort, the lid remains partially opened and the globe turns up and out. This sign is known as Bell’s phenomenon. The incidence is about 20 per 100 000, with a peak in the third decade. The disorder is believed to be viral in origin and is thought to be most often due to occult herpes simplex infection. The facial paresis is often preceded by pain behind the ipsilateral ear, which may resolve. The facial paresis results soon thereafter and may evolve over one to two days, with maximal deficit within the first 72 hours. Eighty to eighty-five per cent of patients recover fully within 3 months. A 7 to 10-day course of oral acyclovir with an oral prednisone taper may shorten the course of the disorder, but studies have not demonstrated a clear-cut benefit to this approach.

    Borrelia burgdorferi, the spirochete responsible for Lyme disease can result in a unilateral, or even bilateral Bell’s palsy. This is often accompanied by an aseptic meningitis, which when combined with facial diplegia is termed Bannwarth’s syndrome. Facial diplegia from Lyme disease is much less common than idiopathic Bell’s palsy and the history should point to deer tick exposure in the outdoor setting at some time directly preceding the onset of symptoms.

    Epstein–Barr virus, West Nile virus, and varicella zoster virus infections do not usually result in a Bell’s palsy. Other conditions that can cause a VII nerve palsy include Guillain–Barré syndrome, sarcoidosis, and facial nerve tumors or metastases.

    Neurology

    B&D Ch. 70

Question 39. E. This question gives a clear description of autism. In autism the child displays a lack of ability to use and read nonverbal gestures, failure to develop appropriate peer relationships, lack of sharing enjoyment with other people, and lack of emotional reciprocity. They also show communication deficits such as delays in language development (not seen in Asperger’s), inability to sustain conversation, repetitive or idiosyncratic use of language, and lack of make-believe play. They exhibit repetitive or stereotyped patterns of behavior such as preoccupation with areas of interest that are abnormal either in intensity or focus, adherence to inflexible ineffective routines, repetitive motor mannerisms, or persistent preoccupation with parts of objects. Some of the above mentioned symptoms appear before three years of age.

    Neurodevelopmental and Pervasive Developmental Disorders

    K&S Ch. 41

Question 40. D. This question points to a diagnosis of rabies virus infection. Rabies virus is transmitted to humans most often by wild animals such as bats, foxes, and raccoons, or by nonimmunized dogs. The incubation period is about one to two months. There is a prodromal period characterized by paresthesias, headache, and fever, which can then progress to generalized neurologic compromise, coma, and death. During the progressive phase of the illness, 80% or more of patients exhibit hydrophobia, which manifests as pharyngeal and nuchal spasms that are triggered by swallowing, smells, tastes, or sounds. These spasms can last up to several minutes in duration. The condition usually progresses to an encephalitis that is often accompanied by high fever that can rise up to 107°F, as well as autonomic hyperactivity, seizures, agitation and psychosis. Treatment begins with postexposure prophylaxis with rabies vaccine and antirabies immunoglobulin in patients who have never received immunization.

    West Nile virus infection is an arbovirus infection that is endemic to many parts of the world. Mosquito bites from the genus Culex are frequent vectors of transmission of the virus to humans. Most infections in humans are asymptomatic. In about 20% of affected patients, the infection presents as a febrile condition after an incubation period of a few days to up to two weeks. Only 1 in 150 patients goes on to develop a meningitis or encephalitis picture. The infection may progress to the development of a demyelinating or axonal neuropathy. Diagnosis is made by detection of IgM antibodies in cerebrospinal fluid, or IgM and IgG antibodies in serum. Treatment is generally supportive. Patients with serious symptoms may respond to intravenous administration of anti-West Nile virus immunoglobulin.

    Acute tetanus results from infection with the bacterium Clostridium tetani. This bacterium releases tetanospasmin, also known as tetanus toxin, which blocks GABA and glycine in the inhibitory interneurons in the spinal cord, and causes the characteristic muscle contractions that are seen in tetanus. The bacterial spores can live for years in dust and soil and when an open wound is inoculated with dirt that contains the spores, they can release the toxin that causes the disease. The toxin is taken up into the anterior horn cells by retrograde axonal transport. Trismus, or lockjaw, is a primary symptom of the disorder in over 75% of cases. Risus sardonicus, a sustained involuntary grimace resulting from uncontrollable facial muscle spasm, is another characteristic of tetanus. Laryngospasm can lead to respiratory compromise and death if the disorder is untreated. Cardiac arrest can also result from dysautonomia. Treatment begins with airway protection and intubation if there is respiratory compromise. Tetanus immune globulin given in a single intramuscular dose of 500 units neutralizes blood-borne toxin. Prevention of the disease by vaccination with anti-tetanus antibody vaccine following a dirty open wound is the cornerstone of disease management.

    Clostridium botulinum is a bacteria that secretes a potent neurotoxin that blocks acetylcholine release at the neuromuscular junction and thus prevents neuromuscular transmission. The bacterium infects humans by its presence in tainted food or by wound contamination from dirt or soil containing the organism. Classic symptoms of botulism include dysphagia, dysarthria, ptosis, and diplopia. These symptoms rapidly progress to limb paralysis and eventually to paralysis of respiratory muscles that can lead to death if the condition is untreated. Gastrointestinal symptoms of nausea, vomiting, and diarrhea, often present with neurologic compromise after a 12–36-hour incubation period following ingestion of the toxin. Infants who consume unpasteurized honey may ingest spores and can present with weak cry, lethargy, floppiness, poor suck, and constipation. Diagnosis can be established by wound or stool culture. Electromyography and nerve conduction studies can reveal characteristic anomalies compatible with presynaptic neuromuscular blockade. Treatment is supportive, particularly with respect to airway protection, and trivalent equine antitoxin administration can reverse the effects of circulating toxin.

    Epstein–Barr virus (EBV) infection can be asymptomatic, or it can present as infectious mononucleosis, with splenomegaly, pharyngitis, and cervical lymphadenopathy. Less than 1% of EBV infections present with neurologic manifestations, which can include meningitis, transverse myelitis, sensory polyneuropathy, or Guillain–Barré syndrome. EBV can be detected in the CSF of patients with AIDS-related primary CNS lymphoma.

    Neurology

    B&D Ch. 53

Question 41. D. The three receptor types associated with glutamate are AMPA, kainate, and N-methyl-D-aspartate (NMDA). Acetylcholine is associated with the nicotinic and muscarinic receptors. Norepinephrine is associated with the alpha 1, alpha 2, and beta receptors. Serotonin is associated with the various 5-HT receptors. GABA is associated with the GABA receptor. Opioids are associated with the mu and delta receptors. Dopamine is associated with the D1, D2, D3, D4 receptors.

    Basic Neuroscience

    K&S Ch. 3

Question 42. B. This vignette depicts a patient with classic manifestations of Pick’s disease. Pick’s disease is one of the so-called frontotemporal dementias (FTDs). The frontal and temporal lobes are preferentially affected and pathological studies reveal localized “knife-edge” atrophy of these lobes, together with ballooned cells and intraneuronal inclusions, termed “Pick’s inclusion bodies”. These inclusions plus the finding of swollen neurons and gliosis have come to be termed Pick-type histology.

    The clinical presentation can manifest either with predominant behavioral disturbance or with progressive language disturbance. In the first type, behavioral changes occur with poor judgment, inability to reason or plan, and impulsivity. Patients can display hyperorality and hypersexuality (similar to that in Klüver–Bucy syndrome) with both behavioral disinhibition and a lack of motivation.

    Treatment is aimed at management of depression and behavioral problems. Antidepressant therapy with SSRI medication can be useful. Atypical antipsychotic agents may help with impulsivity and behavioral anomalies. The condition is invariably irreversible.

    Hirano bodies are eosinophilic neuronal inclusions that can be seen in certain neurodegenerative diseases such as Creutzfeldt–Jakob disease and Alzheimer’s dementia. Lewy bodies are cytoplasmic inclusions found in the substantia nigra and cortex of many patients with idiopathic Parkinson’s disease and in the cerebral cortex of patient’s with Alzheimer’s dementia and diffuse Lewy body disease. Neurofibrillary plaques and tangles are the neuropathologic hallmark of Alzheimer’s dementia. Severe white-matter demyelination can occur in several distinct diseases, particularly in multiple sclerosis, Binswanger’s disease, acute disseminated encephalomyelitis, and progressive multifocal leukoencephalopathy.

    Neurology

    B&D Ch. 66

Question 43. A. It is thought that a child’s temperament at an age as young as 8 months is a function of his genes. At such a young age environmental factors play a small role in the child’s temperament. As the child gets older, environmental factors will play a larger and larger role. Temperament has been examined by researchers and divided into several components. These include activity level, rhythmicity (of hunger, feeding, elimination, etc.), approach or withdrawal, adaptability, intensity of reaction, responsiveness, mood, distractibility, and attention span. There is a genetic component to why these vary among individuals, as well as the influence of parents, consequences of the child’s behavior, and other environmental influences.

    Psychological Theory and Psychometric Testing

    K&S Ch. 2

Question 44. A. The neuropathological hallmark of idiopathic Parkinson’s disease is the finding of characteristic Lewy bodies in surviving neurons in the pars reticulata of the substantia nigra in the midbrain. The substantia nigra itself is depigmented, pale and demonstrates gliosis and neuronal loss. Lewy bodies have a pale halo and an eosinophilic center.

    Hirano bodies are eosinophilic neuronal inclusions that can be seen in certain neurodegenerative diseases such as Creutzfeldt–Jakob disease and Alzheimer’s dementia.

    Amyloid and neuritic plaques are noted in Alzheimer’s dementia. Amyloid may also be found in the cerebrovascular wall. Mesial temporal sclerosis is the pathological hallmark of brain tissue in those with temporal lobe epilepsy. The entorhinal cortex and hippocampi are characterized by neuronal loss and gliosis. Atrophy of the head of the caudate nucleus is found in Huntington’s disease, but it does not correlate with the severity of the disease.

    Neurology

    B&D Chs 66,67&71

Question 45. C. This question gives a description of someone who has schizotypal personality disorder. Schizotypal personality disorder is composed of a pervasive pattern of personal and social deficits characterized by ideas of reference, odd beliefs or magical thinking, unusual perceptual experiences, paranoid ideation, inappropriate affect, eccentric appearance, lack of close friends, and excessive social anxiety that has a paranoid flair.

    Personality Disorders

    K&S Ch. 27

Question 46. B. Fibromyalgia does not present with psychosis. Fibromyalgia is a controversial syndrome characterized by diffuse muscular and soft-tissue pain with multiple tender “trigger” points. The patient must have tenderness to palpation at eleven or more of eighteen specific tender points. Other clinical manifestations can include weakness, paresthesias, sleep disturbance, mood disturbance, headache, fatigue, muscle or joint stiffness, and dizziness. Neurological examination and all laboratory and clinical investigations are usually within normal limits. The cause of the disorder is not known. Treatment involves a good physician–patient relationship, exercise, avoiding a sedentary lifestyle and possibly the use of tricyclic antidepressants.

    Neurology

    B&D Ch. 73

Question 47. C. Pancreatic cancer has been associated with a high rate of depression. Patients present with apathy, decreased energy, and anhedonia. It should be a consideration in the clinician’s mind whenever seeing middle-aged depressed patients. Cancer in general brings about many psychological reactions in patients. Fear of death, fear of abandonment and disfigurement, loss of independence, denial, anxiety, guilt, financial worries, and disruption in relationships all play a role. Of patients with cancer, 50% often have comorbid psychiatric diagnoses with adjustment disorder, major depressive disorder, and delirium being the most common.

    Depressive Disorders

    K&S Ch. 28

Question 48. C. This vignette depicts a classic neurologic emergency: acute epidural spinal cord compression (ESCC). In this case, the causative agent is likely to be a spinal metastasis from the patient’s prostate cancer. Metastases from primary carcinoma of the breast, lung and prostate each make up 20% of the cause of ESCC. Back pain is the most frequent presenting feature in over 80% of cases. The back pain is usually progressive and increases with tumor growth over a period of up to 2 months before the onset of neurological deficits. Motor weakness is noted in about 80% of cases, and it predicts the post-treatment outcome much of the time. Sensory deficits are noted in about 75% of cases. Metastatic ESCC causes a spastic paraparesis or paraplegia with a sensory level that is usually several levels below the actual lesion. Bowel and bladder incontinence, urinary retention, or constipation, are observed in a majority of patients. Sagittal screening MRI imaging of the entire spine is the initial test of choice, if available, because only MRI and myelography can immediately identify and characterize the nature of the lesion and guide treatment. Treatment is undertaken acutely with parenteral corticosteroids and subacutely with radiation therapy. Decompressive laminectomy can be performed, but radiotherapy has of late produced results as good as a surgical approach. Chemotherapy is generally ineffective, because metastases causative of ESCC are usually not chemosensitive.

    Neurology

    B&D Ch. 52

Question 49. D. It is important to be able to distinguish conduct disorder from other childhood psychiatric diagnoses. In conduct disorder the patient shows a pattern whereby the rights of others and societal rules are violated. This presents as bullying other children, using weapons, physical fighting, cruelty to animals, stealing, fire setting, destroying property, truancy, or running away from home. Patients with ADHD can have many behavioral problems, but those problems are a result of inattention, hyperactivity, and impulsivity. ADHD does not show a pattern where the child is intentionally or malevolently being violent or destructive. Any such acts happen as a result of hyperactivity and poor impulse control. In depression children can become very irritable, withdrawn, and not wish to socialize. They may even act out as a result of how badly they are feeling. However this is different than a long-standing pattern of actively trying to carry out violence or do property damage regardless of mood state. In bipolar disorder children may break rules and have behavioral difficulties during manic and depressive episodes. However there will be a clear cycling pattern to their moods (and other symptoms), which corresponds to the times when their behaviors become problematic. One of the other important distinctions to make is between conduct disorder and oppositional defiant disorder (ODD). In ODD there is a pattern of negativistic, hostile, or defiant behavior directed at adults or authority figures. In conduct disorder the negative behavior is directed at all others regardless of whether they are authority figures or not. ODD behaviors are therefore more targeted and have less of a wide-ranging destructive nature than those of conduct disorder.

    Disruptive, Impulse Control, Conduct Disorders, and ADHD

    K&S Ch. 44

Question 50. A. Subacute combined degeneration is the result of a deficiency in vitamin B12 (cobalamin). Cobalamin deficiency manifests as macrocytic anemia, atrophic glossitis, and neurologic deficits. Neurologic symptoms include lesions to the lateral and posterior columns of the spinal cord (subacute combined degeneration), peripheral neuropathy, optic atrophy, and brain lesions. Spinal cord symptoms present as posterior column deficits, which can manifest as upper motor neuron limb weakness, spasticity, and Babinski’s signs. Peripheral neuropathy can present as paresthesias and large fiber sensory impairment (loss of proprioception and vibration sensation). Cerebral symptoms present as behavioral changes, forgetfulness, and in severe cases, dementia and stupor. Sensory ataxia is demonstrated by a positive Romberg sign. There can also be a diffuse hyper-reflexia with absent ankle jerk reflexes. Deficiencies of vitamins B6, niacin, and folic acid are explained elsewhere in this volume.

    Neurology

    B&D Chs 57&76

Question 51. D. Victims of childhood incest often develop depressive feelings mixed with guilt, shame, and a feeling of permanent damage. Teens that undergo sexual abuse often show poor impulse control, self-destructive behavior, and suicidal behavior. Adults abused as children often have post-traumatic stress disorder and dissociative disorders. Incest is most often perpetrated by fathers, stepfathers or older siblings. It is defined as sexual relations between two people who are related or who society has deemed inappropriate to be sexually involved. Most reported cases involve father–daughter incest. In addition to depression, children involved in incest may present to the doctor with complaints of abdominal pain, genital irritation, separation-anxiety disorder, phobias, or school problems.

    Trauma and Stress Related Disorders

    K&S Ch. 32

Question 52. B. This question presents the classic lacunar stroke syndrome called pure motor hemiparesis. This lacunar syndrome classically localizes to the contralateral internal capsule. Two other possible locations of such a lacunar stroke are the contralateral basis pontis or corona radiata. The pure motor stroke presents without cortical deficits. There are no sensory or visual deficits. Aphasia, agnosia and apraxia, are all absent in a pure motor stroke. If both the thalamus and internal capsule are involved, one would expect to see a combined sensory-motor stroke, which manifests not only as a contralateral motor hemiparesis, but also as a contralateral hemisensory loss. The lesion localizes to ischemia in the territory of the small penetrating lenticulostriate arteries that originate off the proximal middle cerebral artery.

    Neurology

    B&D Ch. 51

Question 53. D. The amino acid neurotransmitters include GABA, glutamate, and aspartate. GABA is the main inhibitory neurotransmitter of the brain. Glutamate is the main excitatory neurotransmitter of the brain. The benzodiazepines, barbiturates and several anticonvulsants work through the action of GABA-A and not GABA-B. Lioresal, a muscle relaxant, works through GABA-B. The street drug PCP works primarily through the glutamate receptor. Cocaine works by blocking reuptake of serotonin, norepinephrine, and dopamine. It is also believed to increase glutamate in the nucleus accumbens, accounting for its habit-forming effects. Histamine is a biogenic amine neurotransmitter.

    Basic Neuroscience

    K&S Ch. 3

Question 54. D. Choreoathetosis is not a feature of Parkinson’s disease (PD). The classic cardinal clinical features of idiopathic Parkinson’s disease are akinesia (or bradykinesia), rigidity, resting tremor, and loss of postural reflexes. Tremor is an oscillating 3 to 5 Hz, seen more in the hands where it resembles pill-rolling, but also in the face, chin and the lower extremities. Bradykinesia refers to slowness of voluntary movements that make it difficult for the patient to dress, eat and maintain personal hygiene. Bradykinesia becomes most evident when a patient attempts to perform rapid alternating movements. Rigidity is often manifested as a cogwheeling when the extremities are passively mobilized. Loss of postural reflexes is also a hallmark of idiopathic PD, and is responsible for the many falls sustained by patients with the disease. Other associated clinical features include masked facies, decreased blink, micrographia, seborrhea, weight loss, constipation, and dysphagia. Gait is classically altered and patients walk with “petits pas” (small steps) and a shuffling gait. When they turn around, they turn “en bloc” (without swinging the hips and shoulders, much like a robot). Other autonomic features include orthostatic hypotension, sweating disorder, and urinary frequency. Other symptoms include sleep disturbance, restless legs, fatigue, anxiety, depression, cognitive and behavioral disturbances. These associated features are not present in every case, and are seen less frequently than the cardinal symptoms.

    Neurology

    B&D Ch. 71

Question 55. E. Reading disorder is characterized by reading achievement being substantially below what is expected for the child’s age, intelligence, and education. It interferes with academic achievement and activities of daily living that require reading skills. In the question given, the child’s difficulties are all in situations where she may be asked to read. When children refuse to go to school, separation-anxiety disorder should also be considered. In this case however, the child likes to stay with friends and grandparents, so separation from her parents is not the issue. Children with reading disorder become depressed and demoralized. The diagnosis must be made with a standardized reading test, and pervasive developmental disorders, attention–deficit/hyperactivity disorder, and mental retardation must be ruled out. Treatment consists of understanding the child’s deficits and developing an educational program to remedy them. The child must be given coping strategies such that they are not overwhelmed and discouraged. Coexisting emotional and behavioral problems should also receive treatment.

    Neurodevelopmental and Pervasive Developmental Disorders

    K&S Ch. 39

Question 56. A. Myoclonus is not typically a symptom of botulism toxin poisoning. Botulism results from the ingestion of Clostridium botulinum, a bacterium that extrudes an exotoxin into the circulation and that blocks acetylcholine release at the neuromuscular junction. The bacterium infects humans by its presence in tainted food or by wound contamination from dirt or soil containing the organism. Classic symptoms of botulism include dysphagia, dysarthria, ptosis, diplopia, and urinary retention. Other systemic symptoms include dry mouth and lethargy. Pupillary reflexes are usually impaired. These symptoms rapidly progress to limb paralysis and eventually to paralysis of respiratory muscles that can lead to death if the condition is untreated. Gastrointestinal symptoms of nausea, vomiting, and diarrhea, often present with neurologic compromise after a 12 to 36-hour incubation period following ingestion of the toxin. Infants who consume unpasteurized honey may ingest spores and can present with weak cry, lethargy, floppiness, poor suck, and constipation. Diagnosis can be established by wound or stool culture. Electromyography and nerve conduction studies can reveal characteristic anomalies compatible with presynaptic neuromuscular blockade. Treatment is supportive, particularly with respect to airway protection, and trivalent equine antitoxin administration can help reverse the effects of circulating toxin.

    Neurology

    B&D Ch. 78

Question 57. D. This question gives a clear case of oppositional defiant disorder (ODD). In ODD a child shows a pattern of negativistic, hostile, and defiant behavior directed at adults or authority figures. This behavior may include temper tantrums, arguing with adults, actively defying adults’ requests or rules, deliberately annoying people, blaming others for their mistakes or misbehavior, being easily annoyed by others, being angry and resentful, or being spiteful or vindictive. The child in question should not meet criteria for conduct disorder to be diagnosed with ODD. The primary treatment for ODD is therapy for the child, and parental training to give parents management skills. Often behavioral therapy will be used to reinforce good behavior while ignoring or not reinforcing bad behavior.

    Disruptive, Impulse Control, Conduct Disorders, and ADHD

    K&S Ch. 44

Question 58. E. Balint’s syndrome is a rare stroke syndrome resulting from ischemic lesions to the bilateral parietal-occipital lobes, or the occipital lobes alone. The condition is often a complication of vascular dementia and can occur after a series of strokes to the area involved. It can sometimes be the result of a “top of the basilar syndrome”. The “top of the basilar syndrome” results from rostral basilary artery occlusion that is often embolic in origin. This results in infarction to the midbrain, thalamus and parts of the temporal and occipital lobes. “Top of the basilar syndrome” presents with delirium, peduncular hallucinosis (brainstem-induced visual hallucinations), obtundation, and memory deficits. There may be a gaze palsy, skew deviation of the eyes, ocular bobbing, impaired convergence, or convergence–retraction nystagmus.

    Balint’s syndrome presents with ocular apraxia (the inability to scan extrapersonal space appropriately with the eyes), optic ataxia (dysmetric or saccadic jerks that can impede vision and ocular focus), and deficits in visual attention. Simultanagnosia may also be noted, which is an inability to perceive a scene with multiple parts to it. The patient is only able to see and recognize individual elements of a multi-part scene, and cannot interpret the overall picture. The optic tracts and radiations are usually spared so visual fields are generally normal. Balint’s syndrome can be accompanied by a complete or partial Gerstmann’s syndrome if the dominant parietal lobe is affected in the area of the angular gyrus (Gerstmann’s syndrome is explained elsewhere in this volume). Balint’s syndrome may also result in visuoconstructional apraxia (the inability to put things in their proper place or order). The other answer choices are explained in other questions in this volume, and examination candidates are strongly recommended to review these lobar syndromes as they appear frequently on standardized tests.

    Neurology

    B&D Chs 51&66

Question 59. D. Although there are no specific behaviors that prove that sexual abuse has taken place, children that have been abused often behave in certain patterns. If very young children have detailed knowledge of sexual acts, they have usually witnessed or participated in sexual behavior. They express their sexual knowledge through play and may initiate sexual behavior with other children. Abused children can also be very aggressive. Other signs of sexual abuse include bruising, pain or itching of the genitals, genital or rectal bleeding, recurrent urinary tract infections, or the presence of sexually transmitted disease.

    Trauma and Stress Related Disorders

    K&S Ch. 32

Question 60. D. Anton’s syndrome is considered to be an agnosia characterized by cortical blindness. The lesion localizes to the bilateral occipital lobes, usually due to strokes, particularly in the calcarine cortex (Brodmann’s area 17) and visual association cortex. The arterial territory involved is that of the bilateral posterior cerebral arteries (PCAs). The hallmark of the syndrome is that patients are unaware or deny that they are blind and they confabulate. Patients may also suffer from visual hallucinations.

    Prosopagnosia is the inability to recognize familiar faces. It is associated classically with bilateral occipital-temporal lesions. It is often associated with agraphia and achromatopsia (inability to recognize colors and hues). It is almost always associated with visual field deficits. Aphasia is a disorder of speech and language that can be expressive, receptive, or both. Aphasia occurs following lesions to the dominant hemisphere. Prosopagnosia often occurs following lesions of the occipitotemporal region. The lesion is usually either bilateral or on the right side. Apraxia is the inability to perform simple motor tasks that have been previously learned. Motor apraxia can result from lacunar infarcts to the internal capsule or pons, or from nondominant hemispheric strokes. Hiccups are often the result of phrenic nerve irritation. This is frequently caused by medication side effects such as dopa repletion therapy in Parkinson’s disease.

    Neurology

    B&D Chs 14&66

Question 61. D. Dependent personality disorder (DPD) is the least likely of the answer choices given to present with violence. Patients with dependent personality disorder subordinate their needs to those of others, get others to assume responsibility for major areas of their life, lack self-confidence, and are uncomfortable when alone. They have an intense need to be taken care of that leads them to be clingy, submissive, and fear separation. They can not make decisions without an excessive amount of advice from others. Because of their submissiveness patients with DPD are less likely to become violent than patients with other disorders. People with the other personality disorders listed can be aggressive, outrageous, and at odds with others. Some patients with these disorders may have a clear history of violence. Treatment of dependent personality disorder consists of therapy to modify the patient’s interpersonal interactions, and medication to deal with comorbid anxiety and depression.

    Personality Disorders

    K&S Ch. 27

Question 62. B. Chronic abuse of n-hexane or other hydrocarbon inhalants (glue, paint thinner) can lead to distal sensorimotor polyneuropathy. The first manifestation is usually numbness and distal paresthesia, which is subsequently followed by distal motor weakness. If inhalants continue to be abused, the motor weakness can worsen and can spread to involve the proximal muscles of both the arms and the legs. At low to moderate doses, the inhalants cause dysarthria, uncoordination, euphoria, and relaxation. Higher doses and more chronic exposure can lead to hallucinosis, psychosis, and seizures, as well as more systemic end-organ damage such as renal failure, bone marrow suppression, and hepatic failure.

    Substance Abuse and Addictive Disorders

    B&D Ch. 58

Question 63. D. Delusional disorder is characterized by a fixed false belief that is nonbizarre (i.e., may be possible…such as being poisoned, infected, followed, or having a disease). It is differentiated from schizophrenia by the lack of positive symptoms (no auditory or visual hallucinations, no thought disorganization). Tactile or olfactory hallucinations may be present. Apart from the direct impact of the delusion, the patient does not have a marked impairment in functioning. In the case of somatic delusions, the person believes he has some physical defect or medical condition. Treatment consists of antipsychotics in addition to psychotherapy.

    The other answer choices do not fit the question stem. The patient does not have a neurological deficit, so conversion disorder is incorrect. He does not have auditory or visual hallucinations, and does not have significant negative symptoms and functional impairment, so schizophrenia is unlikely. There is no mention of depressive symptoms. This would not be considered hypochondriasis because hypochondriasis presents with an unreasonable obsession with a perceived medical problem that could be real. This case clearly presents a delusion that has no possible basis in reality, which would push it out of the realm of hypochondriasis.

    Psychotic Disorders

    K&S Ch. 14

Question 64. C. Stimulants such as cocaine typically lower the seizure threshold when abused to intoxication. In moderate doses, cocaine and the stimulants can produce wakefulness, alertness, mood elevation, diminished appetite, and increased performance in certain tasks. The stimulants can also result in psychosis and paranoia. There is also an increased risk of myocardial infarction with cocaine use. Systemic problems such as dehydration, rhabdomyolysis, and hyperthermia can also be noted. The most common neurologic adverse effect of cocaine and the stimulants is headache. In certain cases, the stimulants can produce myoclonus, encephalopathy, and seizures. Cocaine is the stimulant that is most likely to induce seizures. Smoking and intravenous administration of cocaine are more likely to induce seizures than intranasal use. Ischemic and hemorrhagic strokes can also result from cocaine use. Cocaine is the most significant cause of drug-induced stroke and accounts for about 50% of all cases. The mechanism of cocaine-induced stroke is believed to involve vasoconstriction, acute hypertension, and vasospasm.

    Opioid ingestion and intoxication can produce a state of euphoria or dysphoria. Other possible adverse effects include hallucinations, dry mouth, nausea, vomiting, constipation, and urinary retention. Examination usually reveals pupillary constriction during acute intoxication. Autonomic disturbance such as hypotension and hypothermia can also be noted. Seizures are rare with opioid intoxication. Overdose can result in coma and respiratory compromise. Opioids do not usually cause seizures or lower the seizure threshold.

    Phencyclidine (PCP) intoxication produces hallucinosis, dysphoria, and paranoia. Agitation, catatonia, and bizarre behavior are also common. At higher doses, PCP use can lead to stupor and coma. In overdose, rhabdomyolysis may result from agitation and dysautonomia such as fever, hypertension, and sweating. PCP can lower the seizure threshold, but less frequently than cocaine.

    Tetrahydrocannabinol (THC), the primary active ingredient in marijuana, causes euphoria, depersonalization, and relaxation. Other effects of the drug include sleepiness, paranoia, and anxiety. High doses of THC can result in hallucinosis, panic, and even paranoia. Seizures are not generally noted with THC intoxication.

    Alcohol has intoxicating effects that can cause sedation, memory impairment, uncoordination, dysarthria, euphoria, dysphoria, sleepiness, and acute confusion. Alcohol intoxication does not lower the seizure threshold, it in fact is protective against seizures, because of its agonistic effects at the GABA-A receptor. Alcohol withdrawal can lower the seizure threshold because of rapid desaturation of the GABA-A receptor, in much the same way as the benzodiazepines.

    Substance Abuse and Addictive Disorders

    B&D Ch. 58

Question 65. E. Mark, who has major depressive disorder, has the highest likelihood of completing suicide. About 95% of those who attempt or commit suicide have some form of psychiatric disorder. Major depressive disorder accounts for 80% of suicides, schizophrenia accounts for 10%, and dementia or delirium for 5%. Of those who have problems with alcohol, 15% attempt suicide, but the numbers are less than for those with depression. Personality disorders may contribute to suicide attempts, but numbers do not surpass those for depression, and often the two overlap. Mental retardation is not a significant risk factor for suicide. Always keep in mind that the best predictor of future suicidal behavior is past suicidal behavior.

    Management in Psychiatry

    K&S Ch. 34

Question 66. D. Meige’s syndrome denotes the combination of blepharospasm (involuntary eyelid blinking) and oromandibular dystonia. Patients have involuntary contraction of the eyelids as well as the lower facial muscles in the area of the jaw, tongue, or neck. The treatment of choice is chemodenervation of the hyperactive muscles by Botulinum toxin type A injection. The tongue requires careful injection in these cases as it can fall back and occlude the airway if it is weakened too severely. The other answers are simply distracters.

    Neurology

    B&D Ch. 16

Question 67. D. The Tarasoff rule came as a result of the Tarasoff court case. In this case it was decided that a physician or therapist who has reason to believe that a patient might injure or kill someone must notify the potential victim, their family or friends, and the authorities.

    The Durham rule is no longer used, but said that an accused is not responsible if his or her unlawful actions were the result of mental disease or defect.

    The M’Naghten rule comes from British law, stating that a patient is guilty by reason of insanity if they have a mental disease such that they were unaware of the nature, quality, and consequences of their actions, and were incapable of realizing that their actions were wrong.

    Ford vs Wainwright was a case that sustained the need for a patient to be competent in order to be executed. Also worthy of note is that psychiatrists are ethically bound not to participate in state mandated executions in any way.

    Respondeat superior is a legal concept stating that a person at the top of a hierarchy is responsible for the actions of those at the bottom of the hierarchy.

    Forensic Psychiatry

    K&S Ch. 57

Question 68. A. Scanning speech is essentially a form of dysphasia that causes a cerebellar “dysmetria” of the speech pattern. It is also known as ataxic dysarthria. Lesions of the cerebellum such as strokes, degeneration, or tumor can cause this condition. Speech rhythm is usually irregular and choppy. This can also be accompanied by slow, labored speech pattern.

    Neurology

    B&D Ch. 12

Question 69. B. As many as 40% of mothers may experience mood or cognitive symptoms during the postpartum period. Postpartum blues (or maternity blues) is a normal state of sadness, dysphoria, tearfulness, and dependence which may last for several days and is the result of hormonal changes and the stress of being a new mother.

    Postpartum depression is more severe and involves neurovegetative signs and symptoms of depression and potential suicidality.

    Postpartum psychosis can involve hallucinations and delusions, as well as thoughts of infanticide.

    There is no DSM-recognized specific phobia regarding being a parent. The fear of the responsibility associated with parenthood may play a part in postpartum blues or postpartum depression, but is not a distinct entity.

    Depressive Disorders

    K&S Ch. 30

Question 70. D. Aphemia is a motor speech disorder characterized by near muteness with normal reading, writing, and comprehension. Some experts consider aphemia to be the equivalent of pure speech apraxia; however, this remains a controversy. Aphemia is likely to result from lesions to the primary motor cortex or Broca’s area. Patients are first mute, then they become able to speak with hesitancy and phonemic substitutions.

    Aphasia is simply any acquired disorder of language. Aphasias can take multiple forms, including expressive, receptive, conduction, global, or transcortical varieties.

    Agnosia is the inability to recognize and identify objects. Agnosias imply that the patient is able to see, hear, or touch the object to be identified and that the patient does not have a sensory or perceptual deficit that could impair perception of the object.

    Apraxia is the inability to perform a skilled, learned, purposeful motor behavior, in the absence of other deficits that may impair motor functioning. Abulia resembles akinetic mutism. Patients display severe apathy, with affective blunting, amotivation, and immobility. There is an absence of spontaneous speech and movement. Patients retain awareness of their environment.

    Neurology

    B&D Chs 5&12

Question 71. B. The N-methyl-D-aspartate (NMDA) receptor is one of the best known glutamate receptors. It has been found to play a role in learning and memory, as well as in psychopathology. The other glutamate receptors are known as the non-NMDA glutamate receptors. The NMDA receptor allows sodium, potassium, and calcium to pass through. It opens when bound by two glutamate molecules and one glycine molecule at the same time. The receptor can be blocked by physiological concentrations of magnesium, and bound by PCP and PCP-like substances.

    Basic Neuroscience

    K&S Ch. 3

Question 72. C. Stupor refers to a state of unresponsiveness from which arousal only occurs with vigorous and repeated stimulation. It is on the continuum between alertness and coma. Alert patients are awake and in a normal state of arousal. Lethargic patients are sleepy but awake and arousable to alertness with stimulation. Coma implies a state of unarousable unresponsiveness. The persistent vegetative state follows coma. Patients lose cognitive functioning, but retain vegetative, or autonomic functioning (such as cardiac function, respiration, and blood pressure maintenance).

    Neurology

    B&D Ch. 5

Question 73. E. Of the various somatoform disorders, the one with the best prognosis is conversion disorder. The somatoform disorders are characterized by physical symptoms suggestive of a medical condition but are not fully explained by a medical condition or substance abuse. The symptoms are not intentionally produced as in factitious disorders or malingering. Somatization disorder consists of complaints from various organ systems and has a poor prognosis. In hypochondriasis the patient is falsely convinced that he has a serious disease, often based on the misinterpretation of bodily symptoms or functions. Hypochondriasis has a fair to good prognosis. Conversion disorder is the development of one or more neurological deficits that can not be explained by a known medical disorder. Psychological factors are often associated with the onset of the deficit. The prognosis is excellent. Somewhere from 90–100% of patients with conversion disorder are in remission within less than a month. Body dysmorphic disorder is characterized by the false belief or exaggerated perception that a body part is defective. The prognosis is usually poor and the disease is chronic. Pain disorder is marked by pain in one or more sites that is not fully accounted for by a medical condition. The prognosis is guarded and variable.

    Somatic Symptom Disorders

    K&S Ch. 17

Question 74. B. All of these answer choices are at least partly untrue, except for answer choice B. Comatose patients require a head CT scan prior to lumbar puncture (LP), in order to assess if there is a presence of a mass lesion or acute hemorrhage that could result in transtentorial or cerebellar herniation if a LP were to be performed.

    Not every patient with Guillain–Barré syndrome (GBS) should be hospitalized in an intensive care unit. Mild forms of the disease can be monitored in a regular inpatient setting and bedside respiratory peak-flow monitoring can be assessed frequently to screen for acute respiratory compromise that could lead to death. Patients with GBS who deteriorate rapidly, or show signs of poor oxygenation, can be transferred to an intensive care or pulmonary unit for close monitoring, and intubation can be undertaken if deemed clinically necessary. The rule of “20-30-40” is a guide for determining if intensive care unit admission is needed. A vital capacity of less than 20 mL/kg, or a decline by 30% from baseline, or a maximal inspiratory pressure less than 30 cmH2O, or expiratory pressure less than 40 cmH2O, all indicate the need for ICU admission and close monitoring of respiratory status.

    A positive grasp reflex is usually a pathological sign in an adult. It is a normal infantile reflex that is present at birth and usually disappears by 6 months of age. Persistence or redevelopment of the reflex in adulthood would be indicative of frontal lobe pathology and the grasp reflex is considered one of the frontal-release signs.

    Cerebellar hemispheric lesions produce movement deficits that affect the ipsilateral side of the body to the lesion. This is due to the double-crossing of pathways. Ascending cerebellocortical tracts decussate in the midbrain and proceed to the contralateral cortex. Descending corticospinal tracts (the pyramidal tracts) decussate in the medulla to project to the contralateral body.

    Bell’s palsy is believed to be the result of a herpes simplex infection that affects the Gasserian ganglion. Borrelia burgdorferi, the spirochete responsible for Lyme infection, can indeed cause facial diplegia, but is a much rarer infection than that caused by the herpes simplex virus.

    Neurology

    B&D Ch. 76

Question 75. D. The American Psychiatric Association considers it unethical for a psychiatrist to accept a patient’s estate after death. It is considered an exploitation of the therapeutic relationship. It is acceptable to accept a token bequest that you were unaware was in the will when the patient was alive. All of the other acts listed are ethical. Psychiatrists can not participate in executions (often asked on exams). Abandoning patients without arranging for follow-up care is unethical. It is unethical to release information about a patient to another party without the patient’s permission. It is unethical to pay another doctor for sending you referrals. You can rent other doctors space in your office. And it is ethical to charge a fee for supervision.

    Ethics

    K&S Ch. 58

Question 76. A. The amygdala is necessary for the recall of emotional contexts of specific events and the experience of fear, pleasure, or other emotions associated with these events. Declarative or episodic memory (also known as short-term memory) requires the intact functioning of the hippocampus and parahippocampal areas (nucleus basalis of Meynert) of the medial temporal lobe for storage and retrieval of information. The other answers are distracters.

    Basic Neuroscience

    B&D Ch. 6

Question 77. C. Desipramine is the least anticholinergic of the tricyclic antidepressants (TCAs). Anticholinergic side effects are common with patients on TCAs, but tolerance develops over time. Amitriptyline, imipramine, trimipramine, and doxepin are the most anticholinergic. Amoxapine, nortriptyline and maprotiline are less so. Anticholinergic side effects include dry mouth, blurred vision, constipation, and urinary retention. Because of this last side effect desipramine would be the best choice for someone with prostatic hypertrophy. One should always keep in mind the potential for severe anticholinergic effects to lead to a delirium.

    Psychopharmacology

    K&S Ch. 36

Question 78. D. Children are expected to reach developmental milestones at the appropriate age. Standardized examination candidates should memorize these milestones, as questions concerning this material come up on many different examinations. Here are the milestones that need to be remembered:

• 2 months of age: cooing, smiling with social contact, holding head up 45°

• 4 months of age: laughing/squealing, sustaining social contact, grasping objects, weight bearing on legs

• 6 months of age: imitating speech sounds, single syllables, prefers mother, enjoys mirror, transferring objects hand to hand, raking grasp, sitting up with support

• 8 months of age: jabbering, playing peek-a-boo, patty-cake, waving bye-bye, sitting without support, creeping or crawling

• 12 months of age: speech specific to “dada/mama,” playing simple ball games, able to adjust body to dressing, standing alone, able to use thumb-finger pincer grasp

• 14 months of age: one to two-word vocabulary, indicating desires by pointing, hugging parents, walking alone, stooping and recovering

• 18 months of age: six-word vocabulary, able to feed self, walking up stairs while hand is held, imitating scribbling

• 24 months of age: combining words, 250-word vocabulary, helping to undress, listening to picture stories, running well, making circular scribbles, copying a horizontal line

• 30 months of age: knows full name, refers to self as “I”, pretending in play, helping put things away, climbing stairs alternating feet, copying a vertical line

• 36 months of age: counting three objects correctly, knowing age and sex, helping in dressing, riding a tricycle, standing briefly on one foot, copying a circle

• 48 months of age: telling a story, counting four objects, playing with other children, using toilet alone, hopping on one foot, using scissors to cut out pictures, copying a square and a cross

• 60 months of age: naming four colors, counting ten objects, asking about word meanings, domestic role playing, skipping, copying a triangle

    Human Development

    B&D Ch. 7

Question 79. A. This question is a case of exhibitionism. Exhibitionism is a paraphilia in which the patient has a recurrent urge to expose their genitals to strangers. Sexual arousal is brought about by the event. Cases are almost always men exposing themselves to women. Medroxyprogesterone acetate has been shown to be helpful in some cases, and is useful in any sexual disorder in which the patients are extremely hypersexual to the point of being out of control or dangerous. Other drugs such as antipsychotics and antidepressants have not been shown to be particularly useful in such cases. Some patients may improve with the sexual side effects of a serotonin-selective reuptake inhibitor, but it would not be the first choice for treatment.

    Sexual Dysfunctions

    K&S Ch. 21

Question 80. B. The clinical vignette described in this question involves a classic presentation of Hoover’s sign. Hoover’s sign is positive when a patient suspected of a hysterical or psychogenic hemiparesis does not give effort in the contralateral (unaffected) lower extremity when asked to push down on the bed with the paretic (affected) lower extremity. The examiner places a hand under the patient’s heel on the unaffected side to feel if the patient is pushing down towards the bed in an attempt to give a full effort at raising the affected leg. In a real hemiparesis, the patient would be expected to make every effort to brace himself with the unaffected leg while trying to raise the paretic leg.

    Hoffman’s sign is the equivalent to a Babinski’s sign, but it is noted in the upper extremity. The sign is positive when a flick of the distal phalange of the index or middle finger results in an adduction of the ipsilateral thumb. The sign, when present, indicates contralateral corticospinal tract damage that affects the upper extremity.

    Lasègue’s sign is present when straight-leg raising in a recumbent position results in reproduction of pain or paresthesia in the sciatic distribution. The sign can either be ipsilateral or crossed. The positive sign points to sciatic nerve compression due to mechanical interruption of the nerve trajectory, most often due to an intervertebral disk bulge or herniation at the lumbar or sacral level.

    Romberg’s sign is positive when a patient is asked to stand up straight with eyes closed and subsequently loses his balance. The presence of Romberg’s sign usually signals a deficit localizing either to the posterior columns of the spinal cord (i.e., loss of proprioception and/or vibration sensation in the legs), or to cerebellar pathology, or both.

    Gegenhalten refers to “clasp-knife” type rigidity that can be observed in the extremities in several types of disorders that can include stroke, multiple sclerosis, and catatonia.

    Neurology

    B&D Ch. 22

Question 81. E. Clomipramine is one of the tricyclic antidepressants. It has been useful in the treatment of patients with premature ejaculation, depression, panic, obsessive–compulsive disorder, phobias, and pain disorder. It has no use in treating psychosis. It carries with it the cardiac, autonomic, neurologic, sedative, and anticholinergic effects found with many of the tricyclic antidepressants. All tricyclic antidepressants should be avoided during pregnancy.

    Psychopharmacology

    K&S Ch. 35

Question 82. C. The only true emergency in neurology that requires immediate MRI imaging is acute epidural spinal cord compression (ESCC). If MRI is unavailable, the alternate imaging modality of choice is a spinal myelogram. The cause of spinal cord compression can be an intervertebral disk, metastatic carcinoma, or epidural hematoma. Metastases from primary carcinoma of the breast, lung and prostate each make up 20% of the cause of ESCC. Back pain is the most frequent presenting feature in over 80% of cases. The back pain is usually progressive and increases with tumor growth over a period of up to two months before the onset of neurological deficits. Motor weakness is noted in about 80% of cases, and it predicts the post-treatment outcome much of the time. Sensory deficits are noted in about 75% of cases. Metastatic ESCC causes a spastic paraparesis or paraplegia with a sensory level that is usually several levels below the actual lesion. Bowel and bladder incontinence, urinary retention, or constipation, are observed in a majority of patients. Sagittal screening MRI imaging of the entire spine is the initial test of choice, if available, because only MRI and myelography can immediately identify and characterize the nature of the lesion and guide treatment. Treatment is undertaken acutely with parenteral corticosteroids and subacutely with radiation therapy. Decompressive laminectomy can be performed, but radiotherapy has of late produced results as good as a surgical approach. Chemotherapy is generally ineffective, because metastases causative of ESCC are usually not chemosensitive. The other answers are clearly distracters and are discussed individually elsewhere in this volume.

    Neurology

    B&D Ch. 52

Question 83. C. The legal concept of parens patriae allows the state to intervene and act as a surrogate parent for those who are unable to care for themselves or who may harm themselves.

    Actus reus means voluntary conduct. Mens rea means evil intent. Durable power refers to durable power of attorney, where a patient selects who they want to make decisions for them should they become incompetent to make those decisions for themselves. Respondeat superior is the concept that a person at the top of a hierarchy is responsible for the actions of those at the bottom of the hierarchy.

    Forensic Psychiatry

    K&S Ch. 57

Question 84. B. Tiagabine (Gabitril) is the only agent that is a selective GABA reuptake inhibitor. It is FDA-approved as adjunctive treatment of partial complex seizures. Recent warnings have suggested that it can actually worsen or cause seizures. Gabapentin (Neurontin) is a GABA modulating agent that does not directly affect GABA receptors. It is FDA-approved for adjunctive therapy of partial complex seizures and postherpetic neuralgia. Pregabalin (Lyrica) is FDA-approved for adjunctive therapy in partial complex seizures and in neuropathic pain. Its mechanism of action is similar to that of gabapentin: it binds to the alpha-2-delta subunit of the voltage-gated calcium channel. Vigabatrin is not available on the US market. It is an irreversible inhibitor of GABA transaminase (the enzyme that metabolizes GABA) that is approved abroad in the adjunctive therapy of partial complex seizures. Lioresal (Baclofen) is a GABA-B agonist that is approved for spasticity in disorders such as stroke and multiple sclerosis.

    Neurology

    B&D Ch. 67

Question 85. E. The trail-making test is a measure of executive function. The second part of the test consists of drawing lines between a series of letters and numbers in the correct order. For example, the patient would connect A-1-B-2-C-3, etc. The trail-making test is given as part of the Halstead–Reitan Test Battery.

    Psychological Theory and Psychometric Testing

    K&S Ch. 5

Question 86. C. Memantine (Namenda) is an NMDA antagonist that is FDA-approved for the treatment of moderate to severe Alzheimer’s dementia. It has a completely different mechanism of action to donepezil (Aricept), rivastigmine (Exelon), and galantamine (Reminyl), which are inhibitors of acetylcholinesterase that are FDA-approved for treatment of mild to moderate Alzheimer’s dementia. There is now evidence to suggest that concomitant therapy with memantine and one of the cholinesterase inhibitors has measurable clinical advantages over therapy with either class of agent on its own. Neither of these two classes of agent attacks the true pathophysiology of Alzheimer’s dementia, which is the formation of neurofibrillary plaques and tangles in the brain that lead to neuronal cell death. Memantine may reverse the process of apoptosis (preprogrammed cell death) that is believed to be intrinsic to the pathological basis of cell death in Alzheimer’s dementia. Glutamate and NMDA hyperstimulation may lead to premature cell death by promoting calcium influx into neurons and this in turn leads to progression of Alzheimer’s dementia. Memantine may slow disease progress by modulating and lessening the adverse effects of glutamate on the brain. The other answer choices are distracters and need no particular explanation.

    Neurology

    Psychopharmacology

    B&D Ch. 66; also see www.namenda.com

Question 87. A. It is the GABA-A receptor that has binding sites for the benzodiazepines. There are three types of GABA receptors, GABA-A, B, and C. There are no GABA-D or GABA-E receptors. The benzodiazepines increase affinity of the GABA-A receptor for GABA. The GABA-A receptor is a chloride ion channel.

    Psychopharmacology

    K&S Ch. 3

Question 88. A. Vagal nerve stimulation has been FDA-approved since 1997 for adjunctive treatment of refractory partial complex epilepsy in patients over 12 years of age. It recently obtained FDA-approval in intractable major depressive disorder. The modality involves the invasive implantation of an electric stimulating device inside the chest cavity. The stimulator is connected to a wire that wraps around the left vagal nerve (the right vagal nerve is not clinically important in the management of epilepsy). The stimulator is programmed to cycle on for 30 seconds and off for 5 minutes throughout the day. Seizure frequency is typically reduced by about 50%. Side effects can include hoarseness from irritation of the adjacent recurrent laryngeal nerve, and throat tingling and/or coughing during actual stimulation. Vagal nerve stimulation is not as of yet approved for use in any other psychiatric disorder apart from depression.

    Neurology

    B&D Ch. 67

Question 89. D. It is true that some studies have shown the efficacy of using carbamazepine in the treatment of alcohol withdrawal to be equal to that of the benzodiazepines. Carbamazepine is approved in the US for temporal lobe and generalized epilepsy as well as trigeminal neuralgia. It is metabolized by the liver and excreted by the kidneys. It causes its own autoinduction by hepatic enzymes that makes it necessary over time to give more medication to achieve the same blood levels. It also affects the metabolism of several other drugs. Carbamazepine has been associated with a transient decrease in white blood cell count and has been associated with an inhibition of colony-stimulating factor in the bone marrow. It has also been associated with severe blood dyscrasias such as aplastic anemia and agranulocytosis. A benign rash has been found in 10–15% of patients on carbamazepine, and a small percentage of these patients go on to develop serious rashes such as Stevens–Johnson syndrome, exfoliative dermatitis, erythema multiforme, or toxic epidermal necrolysis.

    Psychopharmacology

    K&S Ch. 35

Question 90. E. Auscultation of the head that reveals a bruit is a classic hallmark of an arteriovenous malformation (AVM). Seizures are the presenting symptom of AVMs in about one-quarter to two-thirds of cases. Headaches occur in anywhere from 5 to 35% of cases. Hemorrhage from ruptured AVM tends to be intracerebral in location, rather than subarachnoid as in the case of a ruptured aneurysm. The other distracters are discussed elsewhere in this volume.

    Neurology

    B&D Ch. 51

Question 91. D. The Draw-A-Person test is a projective test. It is administered by telling the patient to draw a person. Then the patient is asked to draw a person of the sex opposite that of the first drawing. The assumption is that the drawing of the patient’s gender is representative of the self in the environment. The level of detail is also correlated with intelligence in children. The Halstead–Reitan battery is used to find the location and effects of certain brain lesions. It is not projective. The Stanford–Binet test is an intelligence test. The Wechsler–Bellvue test is a memory test. The Minnesota Multiphasic Personality Inventory (MMPI) is a self report inventory used to assess personality traits.

    Psychological Theory and Psychometric Testing

    K&S Ch. 5

Question 92. B. The image depicts a classic colloid cyst of the third ventricle. This is a rare condition that presents in middle-aged adults. The cyst is a round, well-circumscribed lesion that is situated in the anterior aspect of the third ventricle. It appears as a “button nose” right in the middle of the ventricular system and this MRI image is a typical example of its appearance. Clinical presentation is that of intermittent headaches that result from increased intracranial pressure because of the ball-valve blockage of the passage of cerebrospinal fluid in the ventricular system. The blockage can even lead to brief intermittent drop attacks in certain patients. These cysts are composed of high concentrations of cholesterol and protein that give them their characteristic hyperintense appearance particularly on T2-weighted MRI brain imaging.

    Neurology

    B&D Ch. 52

image

Question 93. C. Buprenorphine is a mixed opioid agonist/antagonist. It is used for the treatment of heroin addiction as an alternative to methadone. Aripiprazole is a mixed dopamine agonist/antagonist. Naltrexone is an opioid antagonist. Methadone is an opioid agonist. Gabapentin is an anticonvulsant, and is thus unrelated to the opioid receptors.

    Basic Neuroscience

    K&S Ch. 12

Question 94. D. Hallevorden–Spatz syndrome is a rare autosomal recessive disease of childhood onset that presents with the combination of dementia and parkinsonism. The classic neuropathologic hallmark of the disorder is a rusty-brown discoloration of the medial globus pallidus and pars reticulata of the substantia nigra on autopsy. The discoloration is due to the accumulation of iron in the basal ganglia. The disorder is caused by an enzymatic deficiency in cysteine dioxygenase. This leads to increased levels of cysteine in the brain, which chelates iron. Free iron deposits in the basal ganglia and leads to free radical formation, neuronal demise, and ultimately death. Other features of the syndrome include optic atrophy, pigmentary retinopathy, psychomotor retardation and clinical signs of corticospinal tract damage.

    Neurology

    B&D Ch. 66

Question 95. C. Anterograde amnesia is most often associated with alcohol abuse. Keep the Wernicke–Korsakoff syndrome in mind. Wernicke’s encephalopathy is an acute disorder characterized by ataxia, vestibular dysfunction, confusion, and eye movement abnormalities. Korsakoff’s syndrome is a chronic amnestic syndrome that can follow Wernicke’s. It presents with impaired recent memory and anterograde amnesia. The patient may or may not confabulate as well. Long term memory is usually not affected. Treatment for both of these conditions is thiamine administration. If not treated early, permanent damage can take place and Korsakoff’s syndrome can become permanent.

    Substance Abuse and Addictive Disorders

    K&S Ch. 12

Question 96. A. Subacute combined degeneration is the result of a deficiency in vitamin B12 (cobalamin). Cobalamin deficiency manifests as macrocytic anemia, atrophic glossitis, and neurologic deficits. Neurologic symptoms include lesions to the lateral and posterior columns of the spinal cord (subacute combined degeneration), peripheral neuropathy, optic atrophy, and brain lesions. Spinal cord symptoms present as posterior column deficits, which can manifest as upper motor neuron limb weakness, spasticity, and Babinski’s signs. Peripheral neuropathy can present as paresthesias and large fiber sensory impairment (loss of proprioception and vibration sensation). Cerebral symptoms present as behavioral changes, forgetfulness, and in severe cases, dementia and stupor. Sensory ataxia is demonstrated by a positive Romberg sign. There can also be a diffuse hyper-reflexia with absent ankle jerk reflexes. Treatment is undertaken with parenteral B12 injections intramuscularly.

    Neurology

    B&D Ch. 76

Question 97. C. The appropriate action to take in the case presented is to increase the carbamazepine dose and take follow-up levels. This is because carbamazepine has an auto-induction phenomenon whereby it causes the induction of the hepatic enzymes which break it down. Thus, after starting at one dose, the enzymes are induced and break down more carbamazepine, thereby decreasing the serum level. The answer? Give more carbamazepine to bring serum levels back up. The informed clinician knows that this phenomenon will happen, and as such knows the patient is being honest. There is no need to switch to another medication if the carbamazepine was keeping the patient stable. Adding a serotonin-selective reuptake inhibitor would be a good way to flip the patient into overt mania, but not a good way to solve this problem. The patient in question has yet to meet criteria for hospitalization, so that would just be overly aggressive and unwarranted.

    Psychopharmacology

    K&S Ch. 36

Question 98. A. Isoniazid (INH) exposure can cause a vitamin B6 (pyridoxine) deficiency. Most normal adults consume adequate amounts of vitamin B6 in their diets (1.5–2 mg daily). Hydralazine and penicillamine can also cause drug-induced vitamin B6 deficiency. These drugs interfere with vitamin B6 coenzyme activity. Vitamin B6 deficiency results in a distal sensorimotor peripheral neuropathy. Patients can develop distal paresthesias, sensory loss and motor weakness after six months of INH therapy if not on vitamin B6 supplementation. Pyridoxine supplementation of 100 mg daily needs to accompany INH therapy to avoid this untimely deficiency.

    Vitamin A deficiency is rare and can occur with malabsorption syndromes like sprue and biliary atresia. The earliest manifestation is night blindness. Hypervitaminosis A is associated with pseudotumor cerebri which manifests as headache and papilledema. The other answer choices are distracters that are explained elsewhere in this volume.

    Neurology

    B&D Chs 57&76

Question 99. A. The answer to this question is undoubtedly choice A. Although borderline patients make frequent suicidal gestures, they are extremely impulsive. As such it is the psychiatrist’s job to assume that any gesture could be potentially life-threatening and to take the threat seriously. Steps must be taken to protect these patients. Choices B and C are ridiculous and dangerous. Talking about suicide does not increase the risk that patients will try to harm themselves. Isolating them will offer them less support and increase their chances of harming themselves. Choice D is inappropriate because these patients can not be relied on to keep promises when they are in an impulsive, emotionally labile state. Giving these patients a benzodiazepine could potentially disinhibit them further and make it more likely that they would try to harm themselves.

    Management in Psychiatry

    K&S Ch. 33

Question 100. B. West Nile virus infection is an arbovirus (a form of flavivirus) infection that is endemic to many parts of the world. Mosquito bites from the genus Culex are frequent vectors of transmission of the virus to humans. Most infections in humans are asymptomatic. In about 20% of affected patients, the infection presents as a febrile condition after an incubation period of a few days to up to 2 weeks. Only one in 150 patients goes on to develop a meningitis or encephalitis picture. The infection may progress to the development of a demyelinating or axonal neuropathy. Diagnosis is made by detection of IgM antibodies in cerebrospinal fluid, or IgM and IgG antibodies in serum. Treatment is generally supportive. Patients with serious symptoms may respond to intravenous administration of anti-West Nile virus immunoglobulin.

    Arenaviruses are rodent-borne and infect humans when a person comes into contact with infected rodent fecal matter. Lymphocytic choriomeningitis virus and Lassa fever virus are two examples of arenaviruses.

    Filoviruses are represented by the Ebola and Marburg viruses. The reservoir of these viruses is not known. Infection can cause a severe hemorrhagic encephalitis with myositis and muscle pain. Treatment is supportive. Body fluids of these patients are highly contagious.

    Retroviruses are represented by the well-known HIV, HTLV-1 and HTLV-2. The JC virus is a papovavirus and is the causative pathogen of progressive multifocal leukoencephalopathy that can develop in patients with advanced AIDS with low CD4 cell counts. JC virus can be detected in CSF by PCR amplification of its DNA.

    Neurology

    B&D Ch. 53

Question 101. B. You, being the skillful physician that you are, would order a lithium level on this patient! Why? Because the side effects of lithium intoxication include gastrointestinal upset such as nausea, vomiting, and diarrhea. Lithium can cause tremor, nephrogenic diabetes insipidus, acne, muscular weakness, hypothyroidism, weight gain, edema, leukocytosis, psoriasis, hair loss and cardiac dysrhythmias. When toxic it can also cause ataxia, slowed thinking, impaired memory, impaired consciousness, seizures, and death.

    Psychopharmacology

    K&S Ch. 36

Question 102. E. This question asks the exam taker to correlate the neuroanatomical location of a certain type of neurons with the function of the neurotransmitter particular to those neurons. The best answer to this question is choice E, acetylcholine. The basal forebrain is the location of the nucleus basalis of Meynert, which is the structure containing a high density of cholinergic neurons. These neurons project to the limbic system and the cerebral cortex. Alzheimer’s disease is a result of cholinergic neuronal demise predominantly in the nucleus basalis of Meynert.

    Acetylcholine is synthesized from acetylcoenzyme A and choline by the enzyme choline acetyltransferase in the synaptic nerve terminal. Acetylcholine is then stored in vesicles in the synaptic bouton. Once released into the synapse, it is inactivated and metabolized by acetylcholinesterase and the resultant choline is taken back up into the presynaptic terminal for reutilization.

    Acetylcholine is responsible for maintaining short-term memory, attention, executive functioning, and novelty-seeking, which are mediated through the nucleus basalis of Meynert. In Alzheimer’s dementia, acetylcholine is depleted and memory and executive functioning are compromised as a result. The Alzheimer’s agents donepezil, rivastigmine, and galantamine are all acetylcholinesterase inhibitors and can increase levels of circulating acetylcholine in the nucleus basalis and throughout the brain, thereby improving symptoms of dementia to a limited extent. The other neurotransmitters offered in this question are distracters. Each one is explained in other questions in this volume.

    Basic Neuroscience

    K&S Ch. 3

Question 103. E. The tricyclic antidepressants, venlafaxine, bupropion, and nefazodone block the reuptake of norepinephrine (and serotonin in some cases) into the presynaptic neuron. This leads to more norepinephrine in the synaptic cleft. Mirtazapine works by blocking presynaptic alpha-2 receptors, which stops feedback inhibition on the release of norepinephrine into the synaptic cleft. This results in more norepinephrine released into the synapse.

    Psychopharmacology

    K&S Ch. 3

Question 104. D. PCP exerts its hallucinogenic effects by antagonism of N-methyl-D-aspartate (NMDA) receptors, which in turn prevents the influx of calcium ions into neurons. PCP also activates ventral tegmental dopamine which results in the reinforcing qualities of the drug. Tolerance to the physiologic effects of PCP can occur in humans, but dependence and physiologic withdrawal do not usually occur. PCP intoxication produces hallucinosis, dysphoria, and paranoia. Agitation, catatonia, and bizarre behavior are also common. At higher doses, PCP use can lead to stupor and coma. In overdose, rhabdomyolysis may result from agitation and dysautonomia such as fever, hypertension, and sweating. The other answer choices are simply distracters that need no explanation.

    Substance Abuse and Addictive Disorders

    K&S Ch. 12

Question 105. B. The three best benzodiazepines for patients with liver dysfunction are temazepam, oxazepam, and lorazepam. This is an important little fact for the well-prepared test taker to know. They have short half lives, and do not have active metabolites. Other benzodiazepines are less desirable in patients with hepatic dysfunction.

    Psychopharmacology

    K&S Ch. 36

Question 106. C. Aspergillus is a fungus that colonizes in the paranasal sinuses and can cause a hypersensitivity pneumonitis. Infection can originate from the lungs in immunocompromised patients. The fungus has a predilection for invading the posterior circulation and can cause vertebrobasilar strokes. The fungus causes a cerebral vasculitis by invasion of vessel walls. Sinus infection can extend to the brain by contiguous infiltration. Spinal cord compression can result from pulmonary Aspergillosis that extends to the thoracic vertebrae through the epidural space.

    Histoplasma is a fungus that can cause an influenza-like infection with erythematous skin lesions and liver function abnormalities. In fewer than 20% of cases, there is a development of neurologic manifestations in the form of cerebritis, basilar meningitis, or CNS granuloma. Cerebral abscess is also a possible neurologic complication of histoplasmosis infection in about 40% of cases. Meningeal symptoms and signs of headache, fever and neck stiffness can also be noted in the neurologic form of the infection.

    Candida albicans is one of the most common fungal organisms found in the human body. Neurologic infection is rare in immunocompetent hosts. In patients with immune compromise, candidal infection can manifest in the form of intracranial abscesses, vasculitis, and small vessel thrombosis. Candida can form mycotic intracerebral aneurysms that can rupture and cause parenchymal hemorrhage.

    Pseudallecheria boydii is an uncommon fungal pathogen that can infect immunosuppressed patients. Clinical presentation is typically that of a meningitis or multiple brain abscesses.

    Cryptococcus neoformans infection is discussed in detail elsewhere in this volume.

    Neurology

    B&D Ch. 53

Question 107. C. Adverse effects of fluoxetine that set it apart from other serotonin-selective reuptake inhibitors include headache, anxiety, and respiratory complaints. Other side effects include nausea, diarrhea, and insomnia. High blood pressure is found with patients on venlafaxine. Blurred vision can occur from anticholinergic medications. Shuffling gait can occur as a result of parkinsonian side effects of antipsychotics. Loss of consciousness occurs with sedatives.

    Psychopharmacology

    K&S Ch. 36

Question 108. B. The GABA-A receptor (the most predominant GABA receptor) is a chloride channel. Such a useful little fact for the prudent student to know!

    Basic Neuroscience

    K&S Ch. 3

Question 109. D. After making an error in psychodynamic psychotherapy, the best way to proceed is to briefly acknowledge that a mistake was made, and move on, focusing on the patient and their problems. Interpreting the patient’s reaction can be seen as dismissive. It does not address the fact that the therapist made a mistake, not the patient. Ignoring the mistake will contaminate the patient’s transference toward the therapist, potentially making him angry. Giving a long but clear explanation puts too much emphasis on the mistake. The emphasis should be on the patient and his behavior, not the therapist and hers. To profusely apologize is the wrong approach, as it is an over-reaction to a minor mistake. The goal is to acknowledge the mistake and move on, spending as little time focusing on the therapist and her actions and more time on the patient and his actions. Mistakes are a normal part of therapy, as the therapist is human. Dealing with them in a way that maintains boundaries and therapeutic neutrality will be in the best interest of the patient and the therapy.

    Psychotherapy

    K&S Ch. 35

Question 110. C. Identity diffusion is the failure to develop a cohesive self or self-awareness. Do not bother looking the other answer choices up. They are unrelated distracters, some of which are ludicrously unrelated to the question.

    Psychological Theory and Psychometric Testing

    K&S Ch. 2

Question 111. C. Treatment of children with separation-anxiety disorder should be multimodal. It should involve individual therapy for the child, medication to reduce anxiety, family therapy and education, and return to school which is graded if necessary (i.e., start with 1 hour per day, then increase to 2 hours, then 3 hours, etc.). The parental education should focus on giving the child consistent support but maintaining clear boundaries about the child’s avoidant behaviors towards anxiety provoking situations.

    Anxiety Disorders

    K&S Ch. 48

Question 112. D. Naltrexone is an opioid antagonist that is often used as an adjunctive agent for alcohol abuse because it decreases craving and alcohol consumption. It is nowhere near 100% effective and its success is very much dependent on the patient’s desire to stop drinking and the success of concurrent behavioral modification. It is not better than behavioral modification. Naltrexone has nothing to do with dopamine or the GABA receptor.

    Psychopharmacology

    K&S Ch. 36

Question 113. D. The anticholinergic activity of many psychiatric drugs (including tricyclic antidepressants like imipramine) can cause urinary hesitancy, dribbling, and urinary retention. These side effects occur especially with older men who have enlarged prostates. Treatment usually consists of bethanechol 10–30 mg three to four times daily.

    Psychopharmacology

    K&S Ch. 36

Question 114. B. The most important take home point from this question is that bipolar I disorder has equal prevalence for men and women. Major depression is more common in women than in men. There is no correlation between socioeconomic status and frequency of depression. There is a correlation between hypersecretion (not hyposecretion) of cortisol and increased depression. Only about 50% of those with major depressive disorder receive specific treatment.

    Bipolar Disorders

    K&S Ch. 15

Question 115. B. A child who is not speaking should first have her hearing checked. Phonological disorders are characterized by a child’s inability to make age appropriate speech sounds. The child can not be diagnosed if the deficits are being caused by a structural or neurological problem, therefore these things must first be ruled out. Phonological disorder may present as substitutions of one sound for another, or omissions such as leaving the final consonant off of words. The treatment of choice is speech therapy, and recovery can be spontaneous in some children. Speech therapy is indicated if the child can not be understood, is over 8 years old, when self-image and peer relationships are being affected, when many consonants are misarticulated, and when the child is frequently omitting parts of words.

    Diagnostic and Treatment Procedures in Psychiatry

    K&S Ch. 41

Question 116. E. State-dependent learning is the facilitated recall of information in the same internal state or environment in which the information was originally obtained. An example of this is when someone learns a behavior when intoxicated with a drug. Without the drug they can not recall the behavior. When the drug is given to them again they remember the behavior. The other answer choices relate to learning and conditioning. The most important of them have been covered in their own separate questions. Others are just distracters.

    Psychological Theory and Psychometric Testing

    K&S Ch. 4

Question 117. C. This is a case of Wernicke’s encephalopathy. Wernicke’s, and its partner, Korsakoff’s amnesia, are the result of thiamine deficiency often found in alcoholics. Wernicke’s is an acute neurological disorder characterized by ataxia, confusion, vestibular dysfunction, and eye movement impairment. Wernicke’s encephalopathy is reversible with treatment, but if it progresses into Korsakoff’s amnesia, damage may be irreversible. Korsakoff’s presents as impaired recent memory and anterograde amnesia. The treatment for these syndromes is thiamine, first intravenously in the case of an acute Wernicke’s, then orally for as long as 3 to 12 months in the case of Korsakoff’s amnesia. While other answer choices are good ideas, the primary goal is to prevent further brain damage by getting thiamine into the patient immediately.

    Substance Abuse and Addictive Disorders

    K&S Ch. 12

Question 118. B. Niacin is an essential nutrient also called nicotinic acid. Niacin deficiency, termed pellagra, occurs in individuals who consume corn as their main carbohydrate staple. Corn lacks niacin and tryptophan (which can be converted in the body to niacin). Bread is now niacin fortified, which has diminished the widespread problem of pellagra in most countries.

    Pellagra causes the classic triad of the three Ds: dementia, dermatitis and diarrhea. Gastrointestinal problems present as diarrhea, anorexia, and abdominal discomfort. Skin manifestations present as a hyperkeratotic corporal rash over much of the body. The neurological manifestations can include depression, memory impairment, apathy, and irritability. A confusional state may result and may lead to stupor or coma. Oral doses of 50 mg three times daily of nicotinic acid can reverse the symptoms of pellagra.

    The triad of neuropathy, retinopathy, and areflexia can result from a deficiency in vitamin E (α-tocopherol). Other manifestations can include ataxia, loss of proprioception and vibration sensation, nystagmus and external ophthalmoplegia.

    The triad of neuropathy, ataxia, and dementia can result from vitamin B12 (cobalamin) deficiency. Symptoms present as paresthesias of the hands and feet, weakness, gait disturbance, depression, confusion, psychosis, peripheral neuropathy, and loss of position and vibration sensation. Myelopathic symptoms such as spastic paraparesis can occur, as well as visual disturbances as manifested by optic atrophy and visual loss. Treatment is undertaken with parenteral administration of vitamin B12 100 μg daily or 1000 μg twice weekly for 2 weeks.

    Folate deficiency can result in a clinical picture that is similar in presentation to that of B12 deficiency. Elevated serum homocysteine is a surrogate marker for low folate levels. The deficiency can lead to neuropathy and/or spasticity due to spinal cord involvement. Treatment is initially undertaken with 1 mg of folate orally three times daily, followed by a maintenance dose of 1 mg daily. Answer choice A is a nonsense distracter and needs no explanation.

    Neurology

    B&D Ch. 57

Question 119. C. The best way to address a missed therapy session is to use neutral questioning to help explore why it happened. Ignoring the missed appointment is a mistake that can be misunderstood by the patient as the therapist not caring if she shows or not. Getting angry at the patient or punishing the patient is the wrong approach. It will only make the patient angry and less likely to reveal the emotional reasons for why the session was missed, and what meaning that has for the therapeutic relationship. In therapy, the patient’s treatment of the therapist is a reflection of how he or she treats others in life as well. As such, aspects of the patient’s behavior such as missed appointments should be noted and explored.

    Psychotherapy

    K&S Ch. 1

Question 120. D. Tachycardia, not bradycardia, is a symptom of cannabis intoxication. All other answer choices are also symptoms. Orthostatic hypotension is usually only seen with high doses of cannabis.

    Substance Abuse and Addictive Disorders

    K&S Ch. 12

Question 121. E. Akathisia is a subjective feeling of muscular tension caused by antipsychotic medication, which can cause restlessness, pacing, or an inability to stand still. Treatment consists of a beta adrenergic receptor antagonist such as propranolol. Other choices include anticholinergic medications such as benztropine. Benzodiazepines can be useful in some cases.

    Psychopharmacology

    K&S Ch. 36

Question 122. C. Clonidine works by agonist activity at presynaptic alpha-2 receptors. This leads to a decrease in the amount of neurotransmitter released into the synaptic cleft leading to decreased sympathetic tone and decreased arousal. In the case of opioid withdrawal the action of clonidine on the locus ceruleus is thought to be particularly important to the decrease in autonomic symptoms associated with withdrawal. Other answer choices are unrelated distracters.

    Psychopharmacology

    K&S Ch. 36

Question 123. B. Blockade of muscarinic cholinergic receptors is a common side effect of many drugs. Blockade leads to blurred vision, dry mouth, constipation, and difficulty urinating. When there is excessive blockade of this receptor, a patient can develop confusion and delirium. Alzheimer’s disease has been postulated to be in part, the result of too little cholinergic activity. As such, drugs like donepezil block the enzyme acetylcholinesterase (which breaks down acetylcholine) and thereby increase cholinergic activity. This has been shown to be useful in the treatment of dementia. This is the opposite of blockade of muscarinic cholinergic receptors which one would postulate, would worsen the symptoms of Alzheimer’s.

    Psychopharmacology

    K&S Ch. 3

Question 124. A. While all of the lab tests involved in this question can be elevated in alcohol abuse, the most likely test to pick up alcohol abuse is the gamma-glutamyl transferase (GGT). It is elevated in 80% of those with alcohol-related disorders. The other tests are elevated at lower rates than 80% and as such will not pick up as many alcohol disorders as the GGT.

    Laboratory Tests in Psychiatry

    K&S Ch. 12

Question 125. B. Pimozide (Orap) is a dopamine receptor antagonist which has been approved in the US for the treatment of Tourette’s disorder. Haloperidol is also widely used for this indication. In Europe, pimozide is used as an antipsychotic medication for treatment of schizophrenia.

    Psychopharmacology

    K&S Ch. 36

Question 126. C. The gene for amyloid precursor protein is found on chromosome 21. Amyloid precursor protein is broken down to form beta amyloid protein, which is a major component of senile plaques in Alzheimer’s disease.

    Neurocognitive Disorders

    K&S Ch. 10

Question 127. D. Heinz Kohut developed the school of self-psychology. Central to his theories of personality development is the idea that when parents mirror a child’s behavior this functions as a form of empathy, which is necessary for personality development and the formation of healthy self-esteem. When this parental empathy is lacking, the sense of self does not develop properly and personality disorders develop. Patients then need others to fulfill functions that the self would normally handle. Oedipal conflict is a classic part of Freud’s theories in which, greatly simplified, a child competes with the parent of the same sex for the attention of the parent of the opposite sex. The concept of the good enough mother came from Winnicott. He describes a holding environment that develops, where the good enough mother allows the child’s true self to develop. He also gave us the concept of the transitional object. The paranoid–schizoid position and the depressive position are found in the work of Melanie Klein. The paranoid–schizoid position is a view of the world from the perspective of the infant, in which the whole world is split into good and bad elements. The depressive position occurs when the infant is able to view the mother ambivalently as having both positive and negative aspects.

    Human Development

    K&S Ch. 6

Question 128. D. When erythromycin and carbamazepine are given together the carbamazepine levels are increased.

    Psychopharmacology

    K&S Ch. 36

Question 129. D. Valproic acid should be avoided during pregnancy because of its propensity to cause neural tube defects in the developing fetus. It should not be used by nursing mothers, as it is excreted in breast milk. Should the continuation of valproic acid in pregnancy be an absolute necessity, risk of neural tube defects can be reduced by giving the patient 1–4 mg of folic acid per day. However switching to another medication is the best choice.

    Psychopharmacology

    K&S Ch. 36

Question 130. D. Increased appetite, weight gain, and increased sleep make this question a case of atypical depression. The treatment of choice for atypical depression is the monoamine oxidase inhibitors (MAOIs). As such phenelzine is the answer, as it is the only MAOI listed.

    Psychopharmacology

    K&S Ch. 15

Question 131. C. Venlafaxine carries the potential side effect of increasing blood pressure. For this reason a baseline blood pressure should be taken for anyone starting venlafaxine, and regular monitoring of blood pressure is a good idea. Increased blood pressure has been found particularly with doses over 300 mg per day, and lower doses have shown less hypertension. As such, caution must be used when giving this drug to anyone with preexisting hypertension.

    Psychopharmacology

    K&S Ch. 36

Question 132. D. Interpersonal therapy (IPT) was developed to treat depression. It focuses on interpersonal behavior and social interaction. Patients are taught to rate their interactions with others and become aware of their own behavior. The therapist may give direct advice, help make decisions, and clarify conflicts.

    Psychotherapy

    K&S Ch. 35

Question 133. E. Imprinting is the work of Konrad Lorenz. Imprinting implies that an animal has a critical period when it is sensitive to certain stimuli that elicit a specific behavioral response. For example, the baby goose has a period where it “imprints” on its mother and learns to follow her wherever she goes. If a person was the first moving object the goose saw it would imprint on that person and follow that person as if she were the mother. Nikolas Tinbergen conducted experiments both on animal behavior and on humans. He worked on measuring the power of certain stimuli to elicit specific behaviors from animals. He studied displacement activities, where in times when the urge to fight or flee would be equal, the animal would do some other activity to diffuse the tension. Humans also do this in times of stress. He described innate release mechanisms whereby animals have a specific response that is triggered by a releaser. A releaser is an environmental stimulus that prompts the specific response. Tinbergen also worked with human autistic children. He observed both the behavior of autistic children and normal children and postulated that in autistic children certain stimuli that are comforting to a normal child arouse fear in the autistic child. He postulates that this is part of what leads to the behavioral pattern found in autistic children.

    Psychological Theory and Psychometric Testing

    K&S Ch. 4

Question 134. E. The hippocampus is one of the most important structures in the formation of memory. Other areas important to memory include some of the diencephalic nuclei and the basal forebrain. The amygdala also plays a role by rating the emotional content of memories thereby leading to stronger recall of more emotionally charged memories.

    Basic Neuroscience

    K&S Ch. 3

Question 135. E. This is a difficult question unless you know all of the involved scales. It is easier if you know that the brief psychiatric rating scale (BPRS) is a scale used for schizophrenia and psychosis. This is the take-home point. The other scales listed all are mood disorder scales. Will the Montgomery Åsberg scale end up on a standardized test near you? Probably not. But the BPRS probably will, so remember it. It is a good idea to be familiar with the most common psychiatric rating scales such as the BPRS, Hamilton, and GAF (global assessment of functioning – axis V). We will not print the full scales in this text, but it is worth your time to be familiar with them.

    Psychological Theory and Psychometric Testing

    K&S Ch. 9

Question 136. C. Sleep terror disorder is characterized by recurrent episodes of awakening and screaming during the first third of the night. The patient has intense fear, autonomic arousal, sweating, tachycardia, and rapid breathing. The patient is unresponsive to efforts of others to comfort him. There are no dreams recalled and there is amnesia for the event. Small doses of diazepam are often useful to stop the episodes.

    Sleep Wake Disorders

    K&S Ch. 24

Question 137. C. Thioridazine (Mellaril) is one of the older typical antipsychotics that is not used as frequently since the advent of the atypicals. It is very sedating, causes orthostatic hypotension, has anticholinergic side effects, and has low rates of extrapyramidal symptoms. One of its more notable side effects is retrograde ejaculation, in addition to impotence. Patients on thioridazine can be told that retrograde ejaculation is not dangerous, but they will produce milky-white urine following orgasm.

    Psychopharmacology

    K&S Ch. 36

Question 138. C. Donepezil is an acetylcholinesterase inhibitor. All other answer choices are distracters. Donepezil is used to treat mild to moderate Alzheimer’s disease. By blocking acetylcholinesterase, the drug leads to increased acetylcholine in the synaptic cleft which has been proven to slow decline in Alzheimer’s disease.

    Psychopharmacology

    K&S Ch. 36

Question 139. B. Different cultures may present with culture-bound psychiatric syndromes, or different aspects of a certain disorder may be more prevalent in one culture than in another. With regard to depression, Chinese culture often presents with more somatic complaints and less focus on mood symptoms. Very often Chinese patients will come to the primary care physician or emergency room with somatic symptoms that are somewhat non-specific and are found to be driven by an underlying depressive disorder. Cultures have differences in how they view many things, including definitions of what constitutes health and sickness.

    Cultural Issues in Psychiatry

    K&S Ch. 4

Question 140. A. The amphetamines in general exert their effects through the dopaminergic system. However, ecstasy is a “designer amphetamine” which acts through both the dopaminergic and serotonergic systems. Had dopamine been given as an answer choice it would also have been acceptable. The other neurotransmitters listed are unrelated to ecstasy.

    Substance Abuse and Addictive Disorders

    K&S Ch. 12

Question 141. C. The field of child psychiatry developed out of the growth of child guidance centers in the early 1900s. Other answer choices are just distracters. Choice A happened after the early 1900s. Choice B happened before the early 1900s. Choice E took place in 1996 and is a bill relating to HIV testing. Choice D is unrelated to the US, as Freud lived in Vienna.

    History of Psychiatry

    K&S Ch. 36

Question 142. A. Sarcoidosis is a granulomatous disease that affects multiple organ systems. Neurological manifestations occur in about 5% of sarcoidosis patients. Up to 20% of neurologic manifestations present as a peripheral neuropathy. Cranial neuropathies, and in particular facial nerve palsy, are the most common manifestation of neurosarcoidosis, occurring in up to 75% of cases. The treatment of choice is systemic corticosteroid or immunosuppressive therapy. The diagnosis is established clinically, but can often be confirmed by muscle biopsy or an elevated level of angiotensin-converting enzyme (ACE) in the CSF. Other possible neurologic manifestations include cauda equina syndrome, mononeuropathy multiplex, peripheral sensorimotor polyneuropathy, diffuse meningo-encephalitis, uveitis, and polyradiculoneuropathy resembling Guillain–Barré syndrome.

    Neurology

    B&D Chs 49&76

Question 143. D. Trazodone is an antidepressant that is frequently used for insomnia. It is associated with priapism, which is a prolonged erection in the absence of sexual stimulation. Priapism is a potential medical emergency which can be treated by intracavernosal injection of epinephrine. Untreated priapism can also lead to impotence. Patients who start to develop priapism on trazodone should be switched to another medication.

    Psychopharmacology

    K&S Ch. 36

Question 144. C. Low levels of cerebrospinal fluid serotonin are associated with increased aggression. Increased levels of dopamine are associated with increased aggression.

    Basic Neuroscience

    K&S Ch. 4

Question 145. C. Mirtazapine is an antidepressant medication which works by antagonism of presynaptic alpha-2 adrenergic receptors leading to potentiation of serotonergic and noradrenergic neurotransmission. Mirtazapine is sedating, particularly at low doses, which is good for depressed patients with insomnia. It lacks the anticholinergic side effects of the tricyclics, and lacks the anxiogenic side effects of the serotonin-selective reuptake inhibitors. Mirtazapine is also notable for its lack of sexual side effects.

    Psychopharmacology

    K&S Ch. 36

Question 146. D. Sleep changes characteristic of the elderly include both decrease in REM sleep and decrease in slow wave sleep. This is a useful little fact for the test-taker to remember.

    Sleep Wake Disorders

    K&S Ch. 24

Question 147. D. A score of 70 on the global assessment of functioning (GAF) corresponds with “some difficulty in social, occupational, or school functioning, but generally functioning well, has some meaningful interpersonal relationships”. A persistent failure to maintain personal hygiene is a GAF of 10. Major impairment in several areas is a GAF of 40. Superior functioning in all areas is a 100. No friends, unable to keep a job is a GAF of 30. It is a good idea to be familiar with the most common psychiatric rating scales such as the BPRS, Hamilton, and GAF (global assessment of functioning-axis V). We will not print the full scales in this text, but it is worth your time to be familiar with them.

    Diagnostic and Treatment Procedures in Psychiatry

    K&S Ch. 9

Question 148. C. Kleine–Levin syndrome is a rare condition. (But not so rare on standardized tests!) It is marked by periods of hypersomnia with periods of normal sleep in between. During the periods of excessive sleep the patients wake up and experience apathy, irritability, confusion, voracious eating, loss of sexual inhibitions, disorientation, delusions, hallucinations, memory impairment, incoherent speech, excitation, and depression. The onset of the illness usually hits between 10 and 20 years of age, and it goes away by the time the patient is in his forties.

    Disruptive, Impulse Control, Conduct Disorders, and ADHD

    K&S Ch. 24

Question 149. C. David has obsessive–compulsive personality disorder (OCPD). OCPD presents as a pervasive pattern of preoccupation with orderliness, perfectionism, and control. This preoccupation comes at the expense of openness, efficiency, and flexibility. The OCPD patient’s perfectionism interferes with task completion. These patients are inflexible regarding moral and ethical issues. They devote time to work at the expense of leisure activities. They are reluctant to delegate tasks to others. They are characteristically rigid and stubborn. OCPD patients often can not discard old or worn-out objects even when they have no value. They will not give tasks to others without reassurance that the tasks will be done their way. They are miserly in spending and view money as something to be hoarded for catastrophes. David is not presenting with prominent anxiety symptoms, so generalized anxiety disorder is incorrect. He does not have obsessive thoughts, and compulsions to stop those thoughts, so obsessive–compulsive disorder is incorrect. If David had schizoid personality disorder we would see that he had no friends or close contacts and that this does not bother him. In this question David has few friends because he spends all of his time working. If David had avoidant personality disorder he would have a pattern of social inhibition, feelings of inadequacy, and hypersensitivity to negative evaluation. That pattern is not described in this question.

    Personality Disorders

    K&S Ch. 27

Question 150. E. Cognitive behavioral therapy is founded on the principle that people make assumptions that affect their thoughts. Their thoughts then affect their mood. The goal of the therapy is therefore to uncover assumptions and thoughts that may be both faulty and automatic and determine how they contribute to changes in mood. Then the therapy aims to correct these faulty thoughts and stop them from being automatic.

    The other answer choices are all appropriate pieces of psychodynamic therapy and psychoanalysis. They are not however part of cognitive behavioral therapy.

    Psychotherapy

    K&S Ch. 35

 



If you find an error or have any questions, please email us at admin@doctorlib.info. Thank you!